2017 AMC 10/12 B Discussion
Go back to the Math Jam ArchiveAoPS Instructors will discuss problems from the AMC 10/12 B, administered February 15. We will discuss the last 5 problems on each test.
Copyright © 2024 AoPS Incorporated. This page is copyrighted material. You can view and print this page for your own use, but you cannot share the contents of this file with others.
Facilitator: AoPS Staff
copeland
2017-02-16 19:04:31
Are we ready to start?
Are we ready to start?
solzonmars
2017-02-16 19:04:34
Are we doing AMC 10 or 12 first?
Are we doing AMC 10 or 12 first?
copeland
2017-02-16 19:04:41
10. We sort lexicographically.
10. We sort lexicographically.
copeland
2017-02-16 19:05:24
Welcome to the 2016 AMC 10A/12A Math Jam!
Welcome to the 2016 AMC 10A/12A Math Jam!
mumpu2k16
2017-02-16 19:05:26
I AM SO READY FOR THIS!
I AM SO READY FOR THIS!
copeland
2017-02-16 19:05:34
No, wai! Me too!
No, wai! Me too!
ilovemath04
2017-02-16 19:05:38
Let's start!
Let's start!
MathTechFire
2017-02-16 19:05:38
Same
Same
Vfire
2017-02-16 19:05:38
Same
Same
mathchampion1
2017-02-16 19:05:45
2017 B!
2017 B!
jk23541
2017-02-16 19:05:45
i thought it was 2017
i thought it was 2017
copeland
2017-02-16 19:05:51
I try to give you nice things.
I try to give you nice things.
copeland
2017-02-16 19:06:00
Welcome to the 2017 AMC 10A/12A Math Jam!
Welcome to the 2017 AMC 10A/12A Math Jam!
copeland
2017-02-16 19:06:05
OK, one more try.
OK, one more try.
copeland
2017-02-16 19:06:12
Welcome to the 2017 AMC 10B/12B Math Jam!
Welcome to the 2017 AMC 10B/12B Math Jam!
copeland
2017-02-16 19:06:21
So, we can edit all that out in the transcript.
So, we can edit all that out in the transcript.
copeland
2017-02-16 19:06:25
I'm Jeremy Copeland, and I'll be leading our discussion tonight.
I'm Jeremy Copeland, and I'll be leading our discussion tonight.
copeland
2017-02-16 19:06:37
You'll notice devenware. He's responsible for that 2017 typo.
You'll notice devenware. He's responsible for that 2017 typo.
copeland
2017-02-16 19:06:45
I'm the school director here at AoPS. That means when something goes wrong, I either get yelled at or have to yell at someone else. Before AoPS, I was an instructor at MIT, and before that I got my Ph.D. from the University of Chicago. Before that I was an undergrad at Reed College and going back even further, I can't really remember. I used to have hobbies, but I'm a parent now, so those days are all over.
I'm the school director here at AoPS. That means when something goes wrong, I either get yelled at or have to yell at someone else. Before AoPS, I was an instructor at MIT, and before that I got my Ph.D. from the University of Chicago. Before that I was an undergrad at Reed College and going back even further, I can't really remember. I used to have hobbies, but I'm a parent now, so those days are all over.
copeland
2017-02-16 19:06:56
Before we get started I would like to take a moment to explain our virtual classroom procedures to those who have not previously participated in a Math Jam or one of our online classes.
Before we get started I would like to take a moment to explain our virtual classroom procedures to those who have not previously participated in a Math Jam or one of our online classes.
copeland
2017-02-16 19:07:03
The classroom is moderated, meaning that students can type into the classroom, but these comments will not go directly into the room. These comments go to the instructors, who may choose to share your comments with the room.
The classroom is moderated, meaning that students can type into the classroom, but these comments will not go directly into the room. These comments go to the instructors, who may choose to share your comments with the room.
copeland
2017-02-16 19:07:04
This helps keep the class organized and on track. This also means that only well-written comments will be dropped into the classroom, so please take time writing responses that are complete and easy to read.
This helps keep the class organized and on track. This also means that only well-written comments will be dropped into the classroom, so please take time writing responses that are complete and easy to read.
copeland
2017-02-16 19:07:10
There are bunches of students here. As I said, only a fraction of the well-written comments will be passed to the entire group. Please do not take it personally if your comments do not get posted, and please do not complain about it. I expect this Math Jam to be much larger than our typical class, so please be patient with me---there are quite a few of you here tonight!!
There are bunches of students here. As I said, only a fraction of the well-written comments will be passed to the entire group. Please do not take it personally if your comments do not get posted, and please do not complain about it. I expect this Math Jam to be much larger than our typical class, so please be patient with me---there are quite a few of you here tonight!!
copeland
2017-02-16 19:07:18
Also, we won't be going through the math quite as thoroughly as we do in our classes -- I can't teach all the prerequisite material for every problem as we go. Another difference between tonight and our regular online classes is that it is very unlikely that we'll be able to answer every single question you ask. We usually do in our classes, but we have a large number of students tonight! So, please go ahead and ask questions, but also please understand if we aren't able to answer them all!
Also, we won't be going through the math quite as thoroughly as we do in our classes -- I can't teach all the prerequisite material for every problem as we go. Another difference between tonight and our regular online classes is that it is very unlikely that we'll be able to answer every single question you ask. We usually do in our classes, but we have a large number of students tonight! So, please go ahead and ask questions, but also please understand if we aren't able to answer them all!
copeland
2017-02-16 19:07:53
We also have 4 assistants here helping out!
We also have 4 assistants here helping out!
a1b2
2017-02-16 19:07:59
How do you type that fast?
How do you type that fast?
copeland
2017-02-16 19:08:10
(The assistants and I all take turns typing the letters.)
(The assistants and I all take turns typing the letters.)
copeland
2017-02-16 19:08:29
Your assistants for today will be Henrik Boecken (henrikjb), William Wang (willwang123), Joshua Morris (QuantumCat), and Kathryn Lesh (duilka).
Your assistants for today will be Henrik Boecken (henrikjb), William Wang (willwang123), Joshua Morris (QuantumCat), and Kathryn Lesh (duilka).
copeland
2017-02-16 19:08:38
Henrik is an undergraduate at the Massachusetts Institute of Technology studying economics and, of course, math. After college, he plans on teaching at his old high school for a few years. Math contests were the backbone of his high school career, and now he hopes to give back. In his free time, Henrik enjoys Ultimate Frisbee, card games, and reading the Japanese manga One Piece.
Henrik is an undergraduate at the Massachusetts Institute of Technology studying economics and, of course, math. After college, he plans on teaching at his old high school for a few years. Math contests were the backbone of his high school career, and now he hopes to give back. In his free time, Henrik enjoys Ultimate Frisbee, card games, and reading the Japanese manga One Piece.
copeland
2017-02-16 19:08:41
William is a freshman at the University of Pennsylvania. He is a 4-time USA(J)MO qualifier and a USA Physics Olympiad qualifier. In his spare time, he enjoys playing tennis, clarinet, and StarCraft.
William is a freshman at the University of Pennsylvania. He is a 4-time USA(J)MO qualifier and a USA Physics Olympiad qualifier. In his spare time, he enjoys playing tennis, clarinet, and StarCraft.
copeland
2017-02-16 19:08:42
Josh, after getting his Bachelor's and Master's in Physics from UT El Paso and teaching high school physics, decided he loved science and working with students so much he went back to school to work pursue a career in medicine. While he's applying to medical school and going through the interview process, he's helping out here at Art of Problem Solving and keeping his math skills sharp. In his spare time, he loves cooking, running, reading and is (still) slowly teaching himself how to play the piano.
Josh, after getting his Bachelor's and Master's in Physics from UT El Paso and teaching high school physics, decided he loved science and working with students so much he went back to school to work pursue a career in medicine. While he's applying to medical school and going through the interview process, he's helping out here at Art of Problem Solving and keeping his math skills sharp. In his spare time, he loves cooking, running, reading and is (still) slowly teaching himself how to play the piano.
QuantumCat
2017-02-16 19:08:45
Hello everyone!
Hello everyone!
henrikjb
2017-02-16 19:08:50
Hi guys!
Hi guys!
willwang123
2017-02-16 19:08:52
Hello!
Hello!
copeland
2017-02-16 19:09:15
Kathryn decided to be a mathematician when she was twelve, and now teach mathematics in upstate New York. Her son and daughter are in college. Her son studies linguistics, and her daughter is finishing her degree this year and going to grad school in---guess what?--yes, math! In my spare time, she reads, watches Star Trek, and learns Japanese.
Kathryn decided to be a mathematician when she was twelve, and now teach mathematics in upstate New York. Her son and daughter are in college. Her son studies linguistics, and her daughter is finishing her degree this year and going to grad school in---guess what?--yes, math! In my spare time, she reads, watches Star Trek, and learns Japanese.
duilka
2017-02-16 19:09:19
Hi everyone!
Hi everyone!
copeland
2017-02-16 19:09:20
They will be sending you messages to answer questions or offer other help. However, due to the incredibly large size of the session tonight, they may not be able to get to you right away (or at all). Repeating your question over and over may give you warm feelings, but just makes their poor lives harder, so please, only ask your question once and be patient, and please understand that we may not be able to answer all the questions tonight.
They will be sending you messages to answer questions or offer other help. However, due to the incredibly large size of the session tonight, they may not be able to get to you right away (or at all). Repeating your question over and over may give you warm feelings, but just makes their poor lives harder, so please, only ask your question once and be patient, and please understand that we may not be able to answer all the questions tonight.
copeland
2017-02-16 19:09:32
Please also remember that the purpose of this Math Jam is to work through the solutions to AMC problems and learn! "Working through the solutions" includes discussing problem-solving tactics. So please, when a question is posted, do not simply respond with the final answer (since I'll be ignoring those anyway). That's not what we're doing tonight. We're going to work through the problems step-by-step, and comments that skip key steps or jump ahead in the problem, without providing explanation or motivation, won't be posted.
Please also remember that the purpose of this Math Jam is to work through the solutions to AMC problems and learn! "Working through the solutions" includes discussing problem-solving tactics. So please, when a question is posted, do not simply respond with the final answer (since I'll be ignoring those anyway). That's not what we're doing tonight. We're going to work through the problems step-by-step, and comments that skip key steps or jump ahead in the problem, without providing explanation or motivation, won't be posted.
copeland
2017-02-16 19:09:36
Also notice that there will be several cases where we actually find the answer or almost the answer but instead wander off. The goal is always to find a proof that our answer is correct and not just find the answers. Of course on the AMC you should aim to do much less work than this.
Also notice that there will be several cases where we actually find the answer or almost the answer but instead wander off. The goal is always to find a proof that our answer is correct and not just find the answers. Of course on the AMC you should aim to do much less work than this.
copeland
2017-02-16 19:09:54
We will work the last 5 problems from the AMC 10A, then the last 5 problems from the AMC 12A. Two of these problems are the same, 10A Problem 25 and 12A Problem 21. We'll only solve that problem once.
We will work the last 5 problems from the AMC 10A, then the last 5 problems from the AMC 12A. Two of these problems are the same, 10A Problem 25 and 12A Problem 21. We'll only solve that problem once.
copeland
2017-02-16 19:10:06
Should we, get started?
Should we, get started?
mathchampion1
2017-02-16 19:10:19
You wrote "in my spare time, she reads..."
You wrote "in my spare time, she reads..."
copeland
2017-02-16 19:10:20
She only reads when I have free time.
She only reads when I have free time.
zac15SCASD
2017-02-16 19:10:30
YEs!
YEs!
Awesomekid05
2017-02-16 19:10:30
yes!
yes!
shootingstar8
2017-02-16 19:10:30
Yes!
Yes!
Cardinals2014
2017-02-16 19:10:30
Yes
Yes
ILoveChess108
2017-02-16 19:10:30
YES
YES
GeronimoStilton
2017-02-16 19:10:30
Yes!!!!
Yes!!!!
Slacker
2017-02-16 19:10:30
yes!
yes!
techguy2
2017-02-16 19:10:33
AGAIN WITH THE TYPOS
AGAIN WITH THE TYPOS
AthenasEye
2017-02-16 19:10:33
typo again
typo again
copeland
2017-02-16 19:10:39
deven. I'm sick of him.
deven. I'm sick of him.
devenware
2017-02-16 19:10:45
Yeah, my bad.
Yeah, my bad.
copeland
2017-02-16 19:10:53
21. In $\triangle ABC$, $AB = 6$, $AC = 8$, $BC = 10$, and $D$ is the midpoint of $BC$. What is the sum of the radii of the circles inscribed in $\triangle ADB$ and $\triangle ADC$ ?
$\textbf{(A)}\quad \sqrt{5} \qquad \qquad \textbf{(B)}\quad \dfrac{11}{4} \qquad\qquad \textbf{(C)}\quad 2\sqrt{2} \qquad\qquad \textbf{(D)}\quad \dfrac{17}{6} \qquad\qquad\textbf{(E)}\quad 3$
21. In $\triangle ABC$, $AB = 6$, $AC = 8$, $BC = 10$, and $D$ is the midpoint of $BC$. What is the sum of the radii of the circles inscribed in $\triangle ADB$ and $\triangle ADC$ ?
$\textbf{(A)}\quad \sqrt{5} \qquad \qquad \textbf{(B)}\quad \dfrac{11}{4} \qquad\qquad \textbf{(C)}\quad 2\sqrt{2} \qquad\qquad \textbf{(D)}\quad \dfrac{17}{6} \qquad\qquad\textbf{(E)}\quad 3$
copeland
2017-02-16 19:11:03
What's always our first step in a geometry problem?
What's always our first step in a geometry problem?
JJShan26
2017-02-16 19:11:22
diagram
diagram
summitwei
2017-02-16 19:11:22
diagram pl0x
diagram pl0x
reedmj
2017-02-16 19:11:22
Diagram!
Diagram!
letsgomath
2017-02-16 19:11:22
draw a picture!
draw a picture!
a000
2017-02-16 19:11:22
draw a diagram
draw a diagram
RayThroughSpace
2017-02-16 19:11:22
Draw a diagram
Draw a diagram
Skittlesftw
2017-02-16 19:11:22
diagram?
diagram?
Ani10
2017-02-16 19:11:22
diagram
diagram
Reef334
2017-02-16 19:11:22
draw a diagram
draw a diagram
iks92
2017-02-16 19:11:22
draw a picture!
draw a picture!
rspr2001
2017-02-16 19:11:22
draw a diagram
draw a diagram
naman12
2017-02-16 19:11:22
Draw the figure!!!
Draw the figure!!!
zhengyf
2017-02-16 19:11:22
diagrams
diagrams
copeland
2017-02-16 19:11:23
Our first step is always to draw a diagram.
Our first step is always to draw a diagram.
copeland
2017-02-16 19:11:30
But before we even do that, I want to make sure we do it right. What do we know about $\triangle ABC?$
But before we even do that, I want to make sure we do it right. What do we know about $\triangle ABC?$
ilovemath04
2017-02-16 19:11:59
right triangle
right triangle
tootyfail
2017-02-16 19:11:59
its a right triangle
its a right triangle
carlos8
2017-02-16 19:11:59
3-4-5 right triangle
3-4-5 right triangle
blitzkrieg21
2017-02-16 19:11:59
it is a right triangle!
it is a right triangle!
mathwiz0803
2017-02-16 19:11:59
right angle triangle (3-4-5)
right angle triangle (3-4-5)
000libyaclawdruse000
2017-02-16 19:11:59
It's a 3-4-5 right triangle
It's a 3-4-5 right triangle
akaashp11
2017-02-16 19:11:59
3-4-5 Right triangle
3-4-5 Right triangle
yanyu2002
2017-02-16 19:11:59
It's a 3-4-5 Right triangle
It's a 3-4-5 Right triangle
copeland
2017-02-16 19:12:00
We know that $\triangle ABC$ is similar to a 3-4-5 right triangle, and $D$ is the midpoint of the hypotenuse. Okay, let's draw that.
We know that $\triangle ABC$ is similar to a 3-4-5 right triangle, and $D$ is the midpoint of the hypotenuse. Okay, let's draw that.
copeland
2017-02-16 19:12:02
copeland
2017-02-16 19:12:03
(Remember to always draw a large, clear diagram.)
(Remember to always draw a large, clear diagram.)
copeland
2017-02-16 19:12:05
What else should we add to this diagram immediately?
What else should we add to this diagram immediately?
solzonmars
2017-02-16 19:12:28
AD
AD
GeronimoStilton
2017-02-16 19:12:28
$AD$!
$AD$!
Mathaddict11
2017-02-16 19:12:28
segment AD
segment AD
awesome_weisur
2017-02-16 19:12:28
draw AD
draw AD
a1b2
2017-02-16 19:12:28
$AD$
$AD$
ImpossibleCube
2017-02-16 19:12:28
median AD
median AD
mathwiz0803
2017-02-16 19:12:28
AD
AD
ninjataco
2017-02-16 19:12:28
AD
AD
copeland
2017-02-16 19:12:36
It makes sense to draw in $AD$.
It makes sense to draw in $AD$.
copeland
2017-02-16 19:12:37
What else?
What else?
Slacker
2017-02-16 19:12:59
incircle
incircle
tdeng
2017-02-16 19:12:59
The circles
The circles
GeronimoStilton
2017-02-16 19:12:59
The circles!
The circles!
Winston123
2017-02-16 19:12:59
The circles
The circles
BetaCentauri
2017-02-16 19:12:59
maybe the two circles could help
maybe the two circles could help
Derive_Foiler
2017-02-16 19:12:59
the circles?
the circles?
copeland
2017-02-16 19:13:05
Let's also add the incircles of $\triangle ABD$ and $\triangle ADC.$
Let's also add the incircles of $\triangle ABD$ and $\triangle ADC.$
copeland
2017-02-16 19:13:07
copeland
2017-02-16 19:13:16
Okay. What else should we add to this diagram?
Okay. What else should we add to this diagram?
fractal161
2017-02-16 19:13:53
draw radii to the tangents?
draw radii to the tangents?
InYourWildestDreams
2017-02-16 19:13:53
radii
radii
letsgomath
2017-02-16 19:13:53
radii!
radii!
chessapple9
2017-02-16 19:13:53
The radii of the circles that touch the points of tangency
The radii of the circles that touch the points of tangency
naman12
2017-02-16 19:13:53
Radii?
Radii?
thinmint
2017-02-16 19:13:53
The radii
The radii
rick101
2017-02-16 19:13:53
Draw the radii
Draw the radii
PiGuy3141592
2017-02-16 19:13:53
And the radii from each of the sides
And the radii from each of the sides
carfan25
2017-02-16 19:13:53
the radii?
the radii?
copeland
2017-02-16 19:13:56
We care about the radii of the circles, so it makes sense to draw those in. It also always makes sense to draw in radii to tangent points because they give us right angles. I'll draw them all in now, and we'll figure out which ones are useful later.
We care about the radii of the circles, so it makes sense to draw those in. It also always makes sense to draw in radii to tangent points because they give us right angles. I'll draw them all in now, and we'll figure out which ones are useful later.
copeland
2017-02-16 19:13:59
copeland
2017-02-16 19:14:05
Okay, finally, let's add some lengths to our diagram. What lengths do we know?
Okay, finally, let's add some lengths to our diagram. What lengths do we know?
Vfire
2017-02-16 19:14:42
6-8-10
6-8-10
zac15SCASD
2017-02-16 19:14:42
Ab,bc,ac
Ab,bc,ac
BetaCentauri
2017-02-16 19:14:42
the median is half the length of the hypotenuse so AD=5
the median is half the length of the hypotenuse so AD=5
a1b2
2017-02-16 19:14:42
$CD=5$
$CD=5$
awesome_weisur
2017-02-16 19:14:42
AD = CD = BD = 5
AD = CD = BD = 5
jk23541
2017-02-16 19:14:42
AD=CD=BD=5
AD=CD=BD=5
lfwei
2017-02-16 19:14:42
AB = 6, AC = 8, BC = 10
AB = 6, AC = 8, BC = 10
vishwathganesan
2017-02-16 19:14:42
AD = CD = BD = 5, AB = 6, AC = 8
AD = CD = BD = 5, AB = 6, AC = 8
MathTechFire
2017-02-16 19:14:42
Ab =6 Ac = 8 Bc =10
Ab =6 Ac = 8 Bc =10
Magicsm
2017-02-16 19:14:42
AD = 5, CD = 5, DB = 5, AB = 6, AC = 8
AD = 5, CD = 5, DB = 5, AB = 6, AC = 8
copeland
2017-02-16 19:14:45
We know $AB = 6$ and $AC = 8$ because those were given to us.
We know $AB = 6$ and $AC = 8$ because those were given to us.
copeland
2017-02-16 19:14:46
We also know $DC = DB = 5$ because $D$ is the midpoint of $BC$.
We also know $DC = DB = 5$ because $D$ is the midpoint of $BC$.
copeland
2017-02-16 19:14:50
Finally, we know $AD = 5$ because it's the median to the hypotenuse of a right triangle.
Finally, we know $AD = 5$ because it's the median to the hypotenuse of a right triangle.
copeland
2017-02-16 19:14:54
If you didn't already know that, the hypotenuse of a right triangle is the diameter of the circumcircle. Therefore the midpoint of the hypotenuse is the center of the circumcircle, so it is equidistant from all three vertices.
If you didn't already know that, the hypotenuse of a right triangle is the diameter of the circumcircle. Therefore the midpoint of the hypotenuse is the center of the circumcircle, so it is equidistant from all three vertices.
copeland
2017-02-16 19:15:03
copeland
2017-02-16 19:15:15
Alright, finally, we have a beautiful, labeled diagram. I think it's time to get to work.
Alright, finally, we have a beautiful, labeled diagram. I think it's time to get to work.
copeland
2017-02-16 19:15:24
Let's focus on triangle $ADB.$ We have three side lengths, and want to find the inradius. How do those two quantities relate to each other?
Let's focus on triangle $ADB.$ We have three side lengths, and want to find the inradius. How do those two quantities relate to each other?
solzonmars
2017-02-16 19:15:47
Use [Area] = rs
Use [Area] = rs
Derive_Foiler
2017-02-16 19:15:47
Now we can use A=rs?
Now we can use A=rs?
reaganchoi
2017-02-16 19:15:47
$A=rs$
$A=rs$
Awesomekid05
2017-02-16 19:15:47
Now, A=rs?
Now, A=rs?
Magicsm
2017-02-16 19:15:47
use the formula A = rs
use the formula A = rs
Cardinals2014
2017-02-16 19:15:47
A = r*s
A = r*s
checkmatetang
2017-02-16 19:15:47
A=rs
A=rs
geogirl08
2017-02-16 19:15:47
ABC = rs
ABC = rs
mathwizard666
2017-02-16 19:15:47
A=s*r
A=s*r
thedoge
2017-02-16 19:15:47
Area = inradius * semipermeter
Area = inradius * semipermeter
copeland
2017-02-16 19:15:50
We have the formula for the area of a triangle: $[ADB] = rs$ where $r$ is the inradius and $s$ is the semiperimeter.
We have the formula for the area of a triangle: $[ADB] = rs$ where $r$ is the inradius and $s$ is the semiperimeter.
copeland
2017-02-16 19:15:58
If you don't know this one, you can see this quickly in $\triangle ADB$:
If you don't know this one, you can see this quickly in $\triangle ADB$:
copeland
2017-02-16 19:15:59
copeland
2017-02-16 19:16:01
This is three triangles with altitude $r$. The total area is \[\frac{6r}2+\frac{5r}2+\frac{5r}2=\frac{6+5+5}2\cdot r = sr.\]
This is three triangles with altitude $r$. The total area is \[\frac{6r}2+\frac{5r}2+\frac{5r}2=\frac{6+5+5}2\cdot r = sr.\]
copeland
2017-02-16 19:16:11
So the semiperimeter, like I just said, is $s=\dfrac{6+5+5}2=8.$
So the semiperimeter, like I just said, is $s=\dfrac{6+5+5}2=8.$
copeland
2017-02-16 19:16:12
What about the area?
What about the area?
copeland
2017-02-16 19:16:36
(And Heron is not the answer.)
(And Heron is not the answer.)
awesome_weisur
2017-02-16 19:17:10
IT IS HALF THE AREA OF THE RIGHT TRIANGLE
IT IS HALF THE AREA OF THE RIGHT TRIANGLE
reaganchoi
2017-02-16 19:17:10
Both $\frac{8 \cdot 6}{4}=12$
Both $\frac{8 \cdot 6}{4}=12$
thedoge
2017-02-16 19:17:10
it's half of 24, or 12
it's half of 24, or 12
geogirl08
2017-02-16 19:17:10
6*4/2 = 12
6*4/2 = 12
Magicsm
2017-02-16 19:17:10
area is 12, 1/2 the original
area is 12, 1/2 the original
letsgomath
2017-02-16 19:17:10
half of ABC so 12
half of ABC so 12
EulerMacaroni
2017-02-16 19:17:10
$1/2*6*4=12$
$1/2*6*4=12$
kiwitrader123
2017-02-16 19:17:10
12
12
Emathmaster
2017-02-16 19:17:10
12 for both
12 for both
JJShan26
2017-02-16 19:17:10
draw the altitudes to get 4 3-4-5 triangles
draw the altitudes to get 4 3-4-5 triangles
mathman2048
2017-02-16 19:17:10
half the big triangle's area=12
half the big triangle's area=12
copeland
2017-02-16 19:17:12
The area of $ADB$ is half the area of $ABC.$
The area of $ADB$ is half the area of $ABC.$
copeland
2017-02-16 19:17:33
Everything in sight is either a 3-4-5 triangle or a pair of 3-4-5 triangles stuck together.
Everything in sight is either a 3-4-5 triangle or a pair of 3-4-5 triangles stuck together.
copeland
2017-02-16 19:17:47
Heron is a huge pain when you can just multiply lengths.
Heron is a huge pain when you can just multiply lengths.
copeland
2017-02-16 19:18:01
Since the area of $ABC$ is $\dfrac{1}{2} \cdot 8 \cdot 6 = 24$, we know $[ADB] = 12.$
Since the area of $ABC$ is $\dfrac{1}{2} \cdot 8 \cdot 6 = 24$, we know $[ADB] = 12.$
copeland
2017-02-16 19:18:04
So what is the inradius of $ADB$?
So what is the inradius of $ADB$?
mathislife16
2017-02-16 19:18:35
3/2
3/2
PiGuy3141592
2017-02-16 19:18:35
3/2
3/2
ninjataco
2017-02-16 19:18:35
3/2
3/2
vishwathganesan
2017-02-16 19:18:35
12/8 = 3/2
12/8 = 3/2
Awesomekid05
2017-02-16 19:18:35
12/8 = 3/2
12/8 = 3/2
mathchampion1
2017-02-16 19:18:35
$12/8=3/2$
$12/8=3/2$
ilovemath04
2017-02-16 19:18:35
3/2
3/2
High
2017-02-16 19:18:35
3/2
3/2
solzonmars
2017-02-16 19:18:35
4/3
4/3
copeland
2017-02-16 19:18:37
That's $\dfrac{12}{8} = \dfrac{3}{2}$.
That's $\dfrac{12}{8} = \dfrac{3}{2}$.
copeland
2017-02-16 19:18:38
Okay. And what about the inradius of $ADC$?
Okay. And what about the inradius of $ADC$?
mathman2048
2017-02-16 19:19:07
4/3
4/3
ilovemath04
2017-02-16 19:19:07
4/3
4/3
warrenwangtennis
2017-02-16 19:19:07
4/3
4/3
GeronimoStilton
2017-02-16 19:19:07
$\frac{4}{3}$
$\frac{4}{3}$
Slacker
2017-02-16 19:19:07
do the same think
do the same think
ImpossibleCube
2017-02-16 19:19:07
4/3
4/3
a000
2017-02-16 19:19:07
12/9 = 4/3
12/9 = 4/3
lfwei
2017-02-16 19:19:07
12/9 = 4/3
12/9 = 4/3
copeland
2017-02-16 19:19:09
By the exact same argument, that's \[\frac{[\triangle ADC]}{s}=\dfrac{\frac{24}2}{\frac{5+5+8}2}=\dfrac{12}{9}=\dfrac 43.\]
By the exact same argument, that's \[\frac{[\triangle ADC]}{s}=\dfrac{\frac{24}2}{\frac{5+5+8}2}=\dfrac{12}{9}=\dfrac 43.\]
copeland
2017-02-16 19:19:11
So what is our answer?
So what is our answer?
Magicsm
2017-02-16 19:19:32
17/6
17/6
blitzkrieg21
2017-02-16 19:19:32
17/6
17/6
Metal_Bender19
2017-02-16 19:19:32
17/6
17/6
shootingstar8
2017-02-16 19:19:32
17/6
17/6
JJShan26
2017-02-16 19:19:32
D. 17/6
D. 17/6
GeronimoStilton
2017-02-16 19:19:32
$(D)$
$(D)$
Awesomekid05
2017-02-16 19:19:32
4/3 + 3/2 = 17/6 , D
4/3 + 3/2 = 17/6 , D
tdeng
2017-02-16 19:19:32
17/6
17/6
Ani10
2017-02-16 19:19:32
D
D
blitzkrieg21
2017-02-16 19:19:32
17/ 6
17/ 6
copeland
2017-02-16 19:19:34
The answer is $\dfrac 43 + \dfrac 32 = \boxed{\dfrac{17}{6}}$, (D).
The answer is $\dfrac 43 + \dfrac 32 = \boxed{\dfrac{17}{6}}$, (D).
copeland
2017-02-16 19:19:43
Alright. Everyone warmed up now?
Alright. Everyone warmed up now?
Ani10
2017-02-16 19:20:19
wait but why are the two triangles ADC and ADB equal
wait but why are the two triangles ADC and ADB equal
copeland
2017-02-16 19:20:20
That's a great question. Did you see that they're both just a pair of 3-4-5 triangles joined along either the 3-sides or the 4-sides?
That's a great question. Did you see that they're both just a pair of 3-4-5 triangles joined along either the 3-sides or the 4-sides?
copeland
2017-02-16 19:20:47
Now ready for more?
Now ready for more?
Winston123
2017-02-16 19:20:56
Yes
Yes
rspr2001
2017-02-16 19:20:56
yup
yup
AMC10Perfect
2017-02-16 19:20:56
yup
yup
Fives
2017-02-16 19:20:56
yes
yes
techguy2
2017-02-16 19:20:56
YEAAAA BOOOOI
YEAAAA BOOOOI
yanyu2002
2017-02-16 19:20:56
Yes!
Yes!
awesomemaths
2017-02-16 19:20:56
but SURE
but SURE
ILoveChess108
2017-02-16 19:20:56
yes
yes
kunsun
2017-02-16 19:20:56
yup
yup
awesomemaths
2017-02-16 19:20:56
yes
yes
rick101
2017-02-16 19:20:56
Yes!
Yes!
copeland
2017-02-16 19:21:03
22. The diameter $\overline{AB}$ of a circle of radius 2 is extended to a point $D$ outside the circle so that $BD = 3$. Point $E$ is chosen so that $ED =5$ and line $ED$ is perpendicular to line $AD$. Segment $\overline{AE}$ intersects the circle at a point $C$ between $A$ and $E.$ What is the area of $\triangle ABC?$
$\textbf{(A)}\quad \dfrac{120}{37} \qquad \qquad \textbf{(B)}\quad \dfrac{140}{39} \qquad\qquad \textbf{(C)}\quad \dfrac{145}{39} \qquad\qquad \textbf{(D)}\quad \dfrac{140}{37} \qquad\qquad\textbf{(E)}\quad \dfrac{120}{31}$
22. The diameter $\overline{AB}$ of a circle of radius 2 is extended to a point $D$ outside the circle so that $BD = 3$. Point $E$ is chosen so that $ED =5$ and line $ED$ is perpendicular to line $AD$. Segment $\overline{AE}$ intersects the circle at a point $C$ between $A$ and $E.$ What is the area of $\triangle ABC?$
$\textbf{(A)}\quad \dfrac{120}{37} \qquad \qquad \textbf{(B)}\quad \dfrac{140}{39} \qquad\qquad \textbf{(C)}\quad \dfrac{145}{39} \qquad\qquad \textbf{(D)}\quad \dfrac{140}{37} \qquad\qquad\textbf{(E)}\quad \dfrac{120}{31}$
copeland
2017-02-16 19:21:05
Wait, where do we get started again?
Wait, where do we get started again?
Mathaddict11
2017-02-16 19:21:20
Draw a Diagram
Draw a Diagram
StellarG
2017-02-16 19:21:20
Draw a diagram
Draw a diagram
HighQXMoney
2017-02-16 19:21:20
Draw the diagram
Draw the diagram
reedmj
2017-02-16 19:21:20
DIAGRAM!!!
DIAGRAM!!!
wangymin
2017-02-16 19:21:20
diagram
diagram
Reef334
2017-02-16 19:21:20
diagram
diagram
fractal161
2017-02-16 19:21:20
DYE-A-GRAM!
DYE-A-GRAM!
copeland
2017-02-16 19:21:23
A diagram of course. How's this one?
A diagram of course. How's this one?
copeland
2017-02-16 19:21:23
shootingstar8
2017-02-16 19:21:42
Bigger
Bigger
lfwei
2017-02-16 19:21:42
too small
too small
tootyfail
2017-02-16 19:21:42
NOOO bigger
NOOO bigger
pie314159265
2017-02-16 19:21:42
too small ._.
too small ._.
bogstop320
2017-02-16 19:21:42
too small
too small
zac15SCASD
2017-02-16 19:21:42
small
small
BetaCentauri
2017-02-16 19:21:42
uh oh too small
uh oh too small
dr3463
2017-02-16 19:21:42
make it bigger
make it bigger
AthenasEye
2017-02-16 19:21:42
too small
too small
copeland
2017-02-16 19:21:44
TOO SMALL! We won't be able to do any work on this diagram. Remember to draw a large, clear diagram. In fact, I generally suggest you draw a full page diagram if you can.
TOO SMALL! We won't be able to do any work on this diagram. Remember to draw a large, clear diagram. In fact, I generally suggest you draw a full page diagram if you can.
copeland
2017-02-16 19:21:46
copeland
2017-02-16 19:21:49
What pops out at you in this diagram?
What pops out at you in this diagram?
Vfire
2017-02-16 19:22:03
Nice
Nice
copeland
2017-02-16 19:22:04
Why thanks.
Why thanks.
InYourWildestDreams
2017-02-16 19:22:27
ACB is a right triangle
ACB is a right triangle
JJShan26
2017-02-16 19:22:27
ACB is a right triangle and so is ADE
ACB is a right triangle and so is ADE
quartzgirl
2017-02-16 19:22:27
ABC must be a right triangle
ABC must be a right triangle
ImpossibleCube
2017-02-16 19:22:27
ABC is a right triangle
ABC is a right triangle
akaashp11
2017-02-16 19:22:27
<ACB is a right angle!
<ACB is a right angle!
letsgomath
2017-02-16 19:22:27
AC and BC are perpendicular
AC and BC are perpendicular
copeland
2017-02-16 19:22:30
We see that $\angle ACB$ is 90 degrees because $AB$ is the diameter of the circle. What else?
We see that $\angle ACB$ is 90 degrees because $AB$ is the diameter of the circle. What else?
EulerMacaroni
2017-02-16 19:23:05
$\triangle ACB$ and $\triangle ADE$ are siilar
$\triangle ACB$ and $\triangle ADE$ are siilar
Naren12
2017-02-16 19:23:05
triangle ACB and ADE are similar
triangle ACB and ADE are similar
000libyaclawdruse000
2017-02-16 19:23:05
ABC is similar to AED
ABC is similar to AED
reedmj
2017-02-16 19:23:05
5-7-8 right triangle, ACB and ADE are similar
5-7-8 right triangle, ACB and ADE are similar
Ani10
2017-02-16 19:23:05
ACB and ADE similar kek
ACB and ADE similar kek
EulerMacaroni
2017-02-16 19:23:05
$\triangle ACB$ and $\triangle ADE$ are similar and oppositely oriented
$\triangle ACB$ and $\triangle ADE$ are similar and oppositely oriented
Mathaddict11
2017-02-16 19:23:05
ABC and AED are similar triangles!
ABC and AED are similar triangles!
copeland
2017-02-16 19:23:06
Since $\angle ACB = 90^{\circ} = \angle ADE$, and $ACB$ and $ADE$ share $\angle A$, we know these two triangles are similar.
Since $\angle ACB = 90^{\circ} = \angle ADE$, and $ACB$ and $ADE$ share $\angle A$, we know these two triangles are similar.
copeland
2017-02-16 19:23:19
(Don't forget to make the vertices match up when writing similarities.!)
(Don't forget to make the vertices match up when writing similarities.!)
copeland
2017-02-16 19:23:22
So what do we need to do to find the area of $ACB?$
So what do we need to do to find the area of $ACB?$
a000
2017-02-16 19:24:13
ratio of lengths
ratio of lengths
a1b2
2017-02-16 19:24:13
Proportion
Proportion
bluequest
2017-02-16 19:24:13
side ratio^2=area rati
side ratio^2=area rati
quartzgirl
2017-02-16 19:24:13
Find the sides using the ratios first
Find the sides using the ratios first
ilovemath04
2017-02-16 19:24:13
find the ratio
find the ratio
PiGuy3141592
2017-02-16 19:24:13
Find the similarity ratio
Find the similarity ratio
Emathmaster
2017-02-16 19:24:13
Find the side ratio.
Find the side ratio.
copeland
2017-02-16 19:24:16
All we have to do is find the similarity ratio and the area of $ADE.$
All we have to do is find the similarity ratio and the area of $ADE.$
copeland
2017-02-16 19:24:18
One of those is a little easier. What's the area of $ADE?$
One of those is a little easier. What's the area of $ADE?$
lfwei
2017-02-16 19:24:48
35/2
35/2
shootingstar8
2017-02-16 19:24:48
35/2
35/2
letsgomath
2017-02-16 19:24:48
35/2
35/2
reedmj
2017-02-16 19:24:48
35/2
35/2
EulerMacaroni
2017-02-16 19:24:48
$\frac{35}{2}$
$\frac{35}{2}$
Awesomekid05
2017-02-16 19:24:48
35/2
35/2
CornSaltButter
2017-02-16 19:24:48
35/2
35/2
mumpu2k16
2017-02-16 19:24:48
17.5
17.5
blitzkrieg21
2017-02-16 19:24:48
$35/2$
$35/2$
vishwathganesan
2017-02-16 19:24:48
35/2
35/2
Haphazard
2017-02-16 19:24:48
35/2
35/2
copeland
2017-02-16 19:24:53
That's $\dfrac 12 \cdot 7 \cdot 5 = \dfrac{35}{2}.$
That's $\dfrac 12 \cdot 7 \cdot 5 = \dfrac{35}{2}.$
copeland
2017-02-16 19:24:54
Okay, how can we find the similarity ratio?
Okay, how can we find the similarity ratio?
GeronimoStilton
2017-02-16 19:25:46
Find $AE$ so we know $\frac{AB}{AE}$
Find $AE$ so we know $\frac{AB}{AE}$
dr3463
2017-02-16 19:25:46
we know the hypotenuse of ABC and ADE
we know the hypotenuse of ABC and ADE
quartzgirl
2017-02-16 19:25:46
Find the hypotenuse, and we already know the hypotenuse of ABC
Find the hypotenuse, and we already know the hypotenuse of ABC
copeland
2017-02-16 19:25:55
The similarity ratio is $\dfrac{AB}{AE}$.
The similarity ratio is $\dfrac{AB}{AE}$.
copeland
2017-02-16 19:25:57
What is this ratio?
What is this ratio?
ninjataco
2017-02-16 19:26:35
4/sqrt(74)
4/sqrt(74)
MSTang
2017-02-16 19:26:35
4 / sqrt(74)
4 / sqrt(74)
GeronimoStilton
2017-02-16 19:26:35
$\frac{4}{\sqrt{74}}$
$\frac{4}{\sqrt{74}}$
quartzgirl
2017-02-16 19:26:35
4/(sqrt(74))
4/(sqrt(74))
vishwathganesan
2017-02-16 19:26:35
4:sqrt(74)
4:sqrt(74)
abishek99
2017-02-16 19:26:35
4/sqrt(74)
4/sqrt(74)
ILoveChess108
2017-02-16 19:26:35
4/sqrt74
4/sqrt74
Skittlesftw
2017-02-16 19:26:35
4/\sqrt74
4/\sqrt74
mathmagician
2017-02-16 19:26:35
$$4/sqrt(74)$$
$$4/sqrt(74)$$
copeland
2017-02-16 19:26:36
Since $AE = \sqrt{5^2 + 7^2} = \sqrt{74}$ and $AB = 4$, we know the similarity ratio is $\dfrac{4}{\sqrt{74}}.$
Since $AE = \sqrt{5^2 + 7^2} = \sqrt{74}$ and $AB = 4$, we know the similarity ratio is $\dfrac{4}{\sqrt{74}}.$
copeland
2017-02-16 19:26:37
So what is the area of $\triangle ACB?$
So what is the area of $\triangle ACB?$
Reef334
2017-02-16 19:27:24
(D) 140/37
(D) 140/37
Emathmaster
2017-02-16 19:27:24
140/37
140/37
GeronimoStilton
2017-02-16 19:27:24
$\frac{140}{37}$, D
$\frac{140}{37}$, D
warrenwangtennis
2017-02-16 19:27:24
140/37
140/37
JJShan26
2017-02-16 19:27:24
35/2 * 16/74 = 140/37
35/2 * 16/74 = 140/37
tdeng
2017-02-16 19:27:24
8/37*35/2=140/37
8/37*35/2=140/37
Derive_Foiler
2017-02-16 19:27:24
35/2 * 16/74= 140/37
35/2 * 16/74= 140/37
MSTang
2017-02-16 19:27:24
16/74 * 35/2 = 140/37
16/74 * 35/2 = 140/37
summitwei
2017-02-16 19:27:24
140/37
140/37
burunduchok
2017-02-16 19:27:24
140/37
140/37
copeland
2017-02-16 19:27:28
The area is \[ \frac{35}{2} \cdot \left( \frac{4}{\sqrt{74}} \right)^2 = \frac{35 \cdot 16}{148} = \frac{35\cdot 4}{37} = \boxed{\dfrac{140}{37}}.\]
The area is \[ \frac{35}{2} \cdot \left( \frac{4}{\sqrt{74}} \right)^2 = \frac{35 \cdot 16}{148} = \frac{35\cdot 4}{37} = \boxed{\dfrac{140}{37}}.\]
copeland
2017-02-16 19:27:31
Our answer is (D).
Our answer is (D).
copeland
2017-02-16 19:27:40
Any questions or should we soldier on?
Any questions or should we soldier on?
JennyWenDE
2017-02-16 19:27:57
next question
next question
Cardinals2014
2017-02-16 19:27:57
solider
solider
First
2017-02-16 19:27:57
Solider on
Solider on
MathTechFire
2017-02-16 19:27:57
soldier on
soldier on
HighQXMoney
2017-02-16 19:27:57
Soldier on
Soldier on
zew
2017-02-16 19:27:57
keep going!
keep going!
dr3463
2017-02-16 19:27:57
soldier on
soldier on
Fives
2017-02-16 19:28:16
Can we pls have somthing easiar?
Can we pls have somthing easiar?
copeland
2017-02-16 19:28:20
OK, can do!
OK, can do!
copeland
2017-02-16 19:28:24
23. Let \[N=123456789101112\ldots4344\] be the 79-digit number that is formed by writing the integers from 1 to 44 in order, one after the other. What is the remainder when $N$ is divided by 45?
$\textbf{(A)}\quad 1 \qquad \qquad \textbf{(B)}\quad 4 \qquad\qquad \textbf{(C)}\quad 9 \qquad\qquad \textbf{(D)}\quad 18 \qquad\qquad\textbf{(E)}\quad 44$
23. Let \[N=123456789101112\ldots4344\] be the 79-digit number that is formed by writing the integers from 1 to 44 in order, one after the other. What is the remainder when $N$ is divided by 45?
$\textbf{(A)}\quad 1 \qquad \qquad \textbf{(B)}\quad 4 \qquad\qquad \textbf{(C)}\quad 9 \qquad\qquad \textbf{(D)}\quad 18 \qquad\qquad\textbf{(E)}\quad 44$
copeland
2017-02-16 19:28:32
Ooh, ooh! What are we going to use for this problem?
Ooh, ooh! What are we going to use for this problem?
Cardinals2014
2017-02-16 19:29:05
CRT
CRT
First
2017-02-16 19:29:05
CRT
CRT
checkmatetang
2017-02-16 19:29:05
CRT
CRT
CornSaltButter
2017-02-16 19:29:05
Chinese remainder theorem
Chinese remainder theorem
Metal_Bender19
2017-02-16 19:29:05
Chinese remainder theorem
Chinese remainder theorem
burunduchok
2017-02-16 19:29:05
CRT
CRT
dipenm
2017-02-16 19:29:05
Chinese remainder theorem
Chinese remainder theorem
summitwei
2017-02-16 19:29:05
chinese remainder theorem
chinese remainder theorem
MSTang
2017-02-16 19:29:05
CRT!!!
CRT!!!
copeland
2017-02-16 19:29:11
This is a classic Chinese Remainder Theorem problem. If we want to find the remainder after dividing by 45, we can find the remainder after dividing by 5 and the remainder when dividing by 9 and work from there.
This is a classic Chinese Remainder Theorem problem. If we want to find the remainder after dividing by 45, we can find the remainder after dividing by 5 and the remainder when dividing by 9 and work from there.
copeland
2017-02-16 19:29:12
What's the remainder when we divide $N$ by 5?
What's the remainder when we divide $N$ by 5?
BetaCentauri
2017-02-16 19:29:34
4
4
geogirl08
2017-02-16 19:29:34
4
4
techguy2
2017-02-16 19:29:34
4
4
Mathcat1234
2017-02-16 19:29:34
4
4
jeffshen
2017-02-16 19:29:34
4
4
zhengyf
2017-02-16 19:29:34
4
4
Winston123
2017-02-16 19:29:34
4
4
Mathaddict11
2017-02-16 19:29:34
4
4
mathchampion1
2017-02-16 19:29:34
4
4
mathislife16
2017-02-16 19:29:34
4
4
copeland
2017-02-16 19:29:36
The last digit is 4, so
The last digit is 4, so
copeland
2017-02-16 19:29:37
\[N\equiv 4\pmod5.\]
\[N\equiv 4\pmod5.\]
copeland
2017-02-16 19:29:38
What's the remainder when we divide $N$ by 9?
What's the remainder when we divide $N$ by 9?
Cardinals2014
2017-02-16 19:29:56
0
0
quanhui868
2017-02-16 19:29:56
0
0
PiGuy3141592
2017-02-16 19:29:56
0
0
Emathmaster
2017-02-16 19:29:56
0
0
tree3
2017-02-16 19:29:56
0
0
lfwei
2017-02-16 19:29:56
0
0
zhengyf
2017-02-16 19:29:56
0
0
ilovemath04
2017-02-16 19:29:56
0
0
shootingstar8
2017-02-16 19:29:56
0
0
belieber
2017-02-16 19:29:56
0
0
bgu
2017-02-16 19:29:56
0
0
bluequest
2017-02-16 19:29:56
0
0
akaashp11
2017-02-16 19:29:56
0 (mod 9)
0 (mod 9)
summitwei
2017-02-16 19:29:56
0
0
copeland
2017-02-16 19:29:59
We can compute the sum of the digits trickily here. Instead of taking all the digits and adding them, we can take all the numbers and add them:\[N\equiv 12345\cdots4344\equiv1+2+3+\cdots+43+44\pmod{9}.\]
We can compute the sum of the digits trickily here. Instead of taking all the digits and adding them, we can take all the numbers and add them:\[N\equiv 12345\cdots4344\equiv1+2+3+\cdots+43+44\pmod{9}.\]
copeland
2017-02-16 19:30:09
This is an arithmetic progression, so we know the sum is $\dfrac{(1+44)\cdot44}2$, which, whatever it is, is definitely a multiple of 9. So. .
This is an arithmetic progression, so we know the sum is $\dfrac{(1+44)\cdot44}2$, which, whatever it is, is definitely a multiple of 9. So. .
copeland
2017-02-16 19:30:13
\[N\equiv0\pmod9.\]
\[N\equiv0\pmod9.\]
copeland
2017-02-16 19:30:17
What's the remainder when we divide $N$ by 45?
What's the remainder when we divide $N$ by 45?
fractal161
2017-02-16 19:30:48
Has to be $9$
Has to be $9$
prasenjit_hazra
2017-02-16 19:30:48
9
9
Awesomekid05
2017-02-16 19:30:48
9
9
azmath333
2017-02-16 19:30:48
9
9
islander7
2017-02-16 19:30:48
9
9
thetank
2017-02-16 19:30:48
9
9
checkmatetang
2017-02-16 19:30:48
9
9
copeland
2017-02-16 19:30:56
We want to solve these two congruences above. That's best done using guess-and-check.
We want to solve these two congruences above. That's best done using guess-and-check.
copeland
2017-02-16 19:31:00
We want a multiple of 9 that ends in 4 or 9. If you don't see it right away, you should just look at the answer choices!
We want a multiple of 9 that ends in 4 or 9. If you don't see it right away, you should just look at the answer choices!
copeland
2017-02-16 19:31:02
The answer is (C), $\boxed9$.
The answer is (C), $\boxed9$.
copeland
2017-02-16 19:31:36
OK, now back to harder?
OK, now back to harder?
The_Turtle
2017-02-16 19:31:58
YES!
YES!
awesomemaths
2017-02-16 19:31:58
SURE
SURE
zac15SCASD
2017-02-16 19:31:58
Sure
Sure
mumpu2k16
2017-02-16 19:31:58
yep
yep
mathchampion1
2017-02-16 19:31:58
hyperbolas!!!
hyperbolas!!!
copeland
2017-02-16 19:32:06
Hyperbolas? Have you been looking ahead?
Hyperbolas? Have you been looking ahead?
copeland
2017-02-16 19:32:09
24. The vertices of an equilateral triangle lie on the hyperbola $xy=1,$ and a vertex of this hyperbola is the centroid of the triangle. What is the square of the area of the triangle?
$\textbf{(A)}\quad 48 \qquad \qquad \textbf{(B)}\quad 60 \qquad\qquad \textbf{(C)}\quad 108 \qquad\qquad \textbf{(D)}\quad 120 \qquad\qquad\textbf{(E)}\quad 169$
24. The vertices of an equilateral triangle lie on the hyperbola $xy=1,$ and a vertex of this hyperbola is the centroid of the triangle. What is the square of the area of the triangle?
$\textbf{(A)}\quad 48 \qquad \qquad \textbf{(B)}\quad 60 \qquad\qquad \textbf{(C)}\quad 108 \qquad\qquad \textbf{(D)}\quad 120 \qquad\qquad\textbf{(E)}\quad 169$
copeland
2017-02-16 19:32:12
Here's a hyperbola.
Here's a hyperbola.
copeland
2017-02-16 19:32:13
copeland
2017-02-16 19:32:14
Where are the vertices of the hyperbola?
Where are the vertices of the hyperbola?
kunsun
2017-02-16 19:32:49
1,1 and -1,-1
1,1 and -1,-1
InYourWildestDreams
2017-02-16 19:32:49
(1, 1) (-1, -1)
(1, 1) (-1, -1)
a000
2017-02-16 19:32:49
(1,1) and (-1,-1)
(1,1) and (-1,-1)
joslin94
2017-02-16 19:32:49
(1,1) and (-1,-1)
(1,1) and (-1,-1)
a1b2
2017-02-16 19:32:49
$(\pm 1, \pm 1)$
$(\pm 1, \pm 1)$
PiGuy3141592
2017-02-16 19:32:49
(1,1) and (-1,-1)
(1,1) and (-1,-1)
reedmj
2017-02-16 19:32:49
$(1,1) \text{ and } (-1,-1)$
$(1,1) \text{ and } (-1,-1)$
quartzgirl
2017-02-16 19:32:49
(1, 1), (-1, -1)
(1, 1), (-1, -1)
mathwrite
2017-02-16 19:32:49
(1,1) and (-1,-1)
(1,1) and (-1,-1)
kiwitrader123
2017-02-16 19:32:49
(!,1), (-1,-1)
(!,1), (-1,-1)
copeland
2017-02-16 19:32:51
The vertices are the closest points on the branches. They're the points where the line through the foci intersect the parabola. Since the hyperbola is symmetric across $y=x$, the foci are on $y=x$, so the vertices are at $(1,1)$, and $(-1,-1)$.
The vertices are the closest points on the branches. They're the points where the line through the foci intersect the parabola. Since the hyperbola is symmetric across $y=x$, the foci are on $y=x$, so the vertices are at $(1,1)$, and $(-1,-1)$.
copeland
2017-02-16 19:33:02
Now we want an equilateral triangle with center at $(1,1)$. They use the word "centroid", but which center is more useful to think about?
Now we want an equilateral triangle with center at $(1,1)$. They use the word "centroid", but which center is more useful to think about?
mathfun5
2017-02-16 19:33:37
Circumcenter
Circumcenter
zac15SCASD
2017-02-16 19:33:37
circumcenter
circumcenter
a1b2
2017-02-16 19:33:37
Circumcenter
Circumcenter
letsgomath
2017-02-16 19:33:37
circumcenter?
circumcenter?
mathchampion1
2017-02-16 19:33:37
circumcenter!
circumcenter!
tdeng
2017-02-16 19:33:37
circumcenter
circumcenter
ninjataco
2017-02-16 19:33:37
circumcenter
circumcenter
copeland
2017-02-16 19:33:39
Why?
Why?
MSTang
2017-02-16 19:35:06
the circumcircle must hit the other vertex of the hyperbola
the circumcircle must hit the other vertex of the hyperbola
mathfun5
2017-02-16 19:35:06
contrsuct the circumcircle! It then passes through the hyperbola
contrsuct the circumcircle! It then passes through the hyperbola
copeland
2017-02-16 19:35:07
The circumcenter contains the vertices, so the circumcenter will intersect the hyperbola at the vertices (and maybe elsewhere).
The circumcenter contains the vertices, so the circumcenter will intersect the hyperbola at the vertices (and maybe elsewhere).
copeland
2017-02-16 19:35:24
In an equilateral triangle, the centroid is also the circumcenter so we can use that. Let's think about circles with circumcenter $(1,1)$.
In an equilateral triangle, the centroid is also the circumcenter so we can use that. Let's think about circles with circumcenter $(1,1)$.
copeland
2017-02-16 19:35:33
Now we just draw a circle with center $(1,1)$ that intersects the hyperbola. Here are a few:
Now we just draw a circle with center $(1,1)$ that intersects the hyperbola. Here are a few:
copeland
2017-02-16 19:35:34
copeland
2017-02-16 19:35:43
What do you notice?
What do you notice?
shootingstar8
2017-02-16 19:35:58
Woah!
Woah!
Emathmaster
2017-02-16 19:35:58
Yellow.
Yellow.
copeland
2017-02-16 19:36:00
Go go hsv.
Go go hsv.
copeland
2017-02-16 19:36:38
(hsv is a way to define colors. Nevermind.)
(hsv is a way to define colors. Nevermind.)
mathislife16
2017-02-16 19:36:50
the yellow one hits the other vertex
the yellow one hits the other vertex
ninjataco
2017-02-16 19:36:50
intersects in 2, 3, or 4 points
intersects in 2, 3, or 4 points
The_Turtle
2017-02-16 19:36:50
The red circle intersects the graph twice, the yellow one intersects three times, and the rest intersect 4 times
The red circle intersects the graph twice, the yellow one intersects three times, and the rest intersect 4 times
zac15SCASD
2017-02-16 19:36:50
only one hits exactly 3 points (yellow)
only one hits exactly 3 points (yellow)
techguy2
2017-02-16 19:36:50
we want the one that touches in only three places
we want the one that touches in only three places
tree3
2017-02-16 19:36:50
the yellow circle works
the yellow circle works
Zekrom
2017-02-16 19:36:50
it must intersect the other vertex because otherwise, there are four or two vertices on the hyperbola
it must intersect the other vertex because otherwise, there are four or two vertices on the hyperbola
copeland
2017-02-16 19:37:03
So, it's not a priori bad if the circle intersects 4 times instead of 3.
So, it's not a priori bad if the circle intersects 4 times instead of 3.
copeland
2017-02-16 19:37:13
However, why are those 4-intersection guys unsatisfying?
However, why are those 4-intersection guys unsatisfying?
rafayaashary1
2017-02-16 19:37:58
Symmetry
Symmetry
Derive_Foiler
2017-02-16 19:37:58
not equilateral
not equilateral
vishwathganesan
2017-02-16 19:37:58
not symmetrical
not symmetrical
dipenm
2017-02-16 19:37:58
they can't be equilateral
they can't be equilateral
techguy2
2017-02-16 19:37:58
no three points are equidistant
no three points are equidistant
mathwiz0803
2017-02-16 19:37:58
they don't form equilateral triangles
they don't form equilateral triangles
GeronimoStilton
2017-02-16 19:37:58
Because they give us two possible equilateral triangles, and they're not symmetric.
Because they give us two possible equilateral triangles, and they're not symmetric.
The_Turtle
2017-02-16 19:37:58
No subset of their points form an equilateral triangle
No subset of their points form an equilateral triangle
AlcumusGuy
2017-02-16 19:37:58
cannot be equilateral
cannot be equilateral
copeland
2017-02-16 19:38:01
These are usually quadrilaterals.
These are usually quadrilaterals.
copeland
2017-02-16 19:38:03
These all have $y=x$ as an axis of symmetry, so they're actually isosceles trapezoids.
These all have $y=x$ as an axis of symmetry, so they're actually isosceles trapezoids.
copeland
2017-02-16 19:38:22
Three vertices of an isosceles trapezoid can't be the vertices of an equilateral triangle. The axis of symmetry would go through one of the sides of the triangle and through the other vertex. That would give us a degenerate trapezoid.
Three vertices of an isosceles trapezoid can't be the vertices of an equilateral triangle. The axis of symmetry would go through one of the sides of the triangle and through the other vertex. That would give us a degenerate trapezoid.
copeland
2017-02-16 19:38:33
So the only triangle we could get has to come from the circle that's tangent to the other branch:
So the only triangle we could get has to come from the circle that's tangent to the other branch:
copeland
2017-02-16 19:38:34
copeland
2017-02-16 19:38:46
I sure hope that's equilateral.
I sure hope that's equilateral.
The_Turtle
2017-02-16 19:39:27
How can we tell?
How can we tell?
pie314159265
2017-02-16 19:39:27
we can assume
we can assume
yayups
2017-02-16 19:39:27
by the problem statement, it must be
by the problem statement, it must be
BLCRAFT
2017-02-16 19:39:27
xD how do we know?
xD how do we know?
yanyu2002
2017-02-16 19:39:27
How can you just hope it's equilateral?
How can you just hope it's equilateral?
GeronimoStilton
2017-02-16 19:39:27
It doesn't matter because it's the only possible triangle.
It doesn't matter because it's the only possible triangle.
copeland
2017-02-16 19:39:34
These are all fine emotions to be having right now. I have them all, too.
These are all fine emotions to be having right now. I have them all, too.
copeland
2017-02-16 19:39:46
What say we take a moment to check, actually? You should definitely not do that on the test, but we have the luxury of doing it now.
What say we take a moment to check, actually? You should definitely not do that on the test, but we have the luxury of doing it now.
copeland
2017-02-16 19:40:01
I'm going to spew math at you for a moment, because this is pretty hard.
I'm going to spew math at you for a moment, because this is pretty hard.
Ani10
2017-02-16 19:40:05
use coordinate geo?
use coordinate geo?
copeland
2017-02-16 19:40:12
What's a generic point on the hyperbola look like?
What's a generic point on the hyperbola look like?
Derive_Foiler
2017-02-16 19:40:32
(x,1/x)
(x,1/x)
no_name
2017-02-16 19:40:32
(a, 1/a)
(a, 1/a)
akaashp11
2017-02-16 19:40:32
(x,1/x)
(x,1/x)
mathfun5
2017-02-16 19:40:32
(a,1/a)
(a,1/a)
AlcumusGuy
2017-02-16 19:40:32
(x, 1/x)
(x, 1/x)
abishek99
2017-02-16 19:40:32
(a,1/a)
(a,1/a)
zac15SCASD
2017-02-16 19:40:32
(x, 1/x)
(x, 1/x)
PiGuy3141592
2017-02-16 19:40:32
(x,1/x)
(x,1/x)
summitwei
2017-02-16 19:40:32
(x,1/x)
(x,1/x)
burunduchok
2017-02-16 19:40:32
$(x, \frac{1}{x})$
$(x, \frac{1}{x})$
000libyaclawdruse000
2017-02-16 19:40:32
(x, 1/x)
(x, 1/x)
copeland
2017-02-16 19:40:37
A generic point looks like $P=\left(x,\frac1x\right)$.
A generic point looks like $P=\left(x,\frac1x\right)$.
copeland
2017-02-16 19:40:38
The red circle has squared radius $r^2=8$. So a point on the circle looks like \[(x-1)^2+(y-1)^2=8.\]
The red circle has squared radius $r^2=8$. So a point on the circle looks like \[(x-1)^2+(y-1)^2=8.\]
copeland
2017-02-16 19:40:43
The intersection points of the circle with the hyperbola solve\[\left(x-1\right)^2+\left(\frac1x-1\right)^2=8.\]
The intersection points of the circle with the hyperbola solve\[\left(x-1\right)^2+\left(\frac1x-1\right)^2=8.\]
copeland
2017-02-16 19:40:48
We can expand that to get \[x^2+\frac1{x^2}-2\left(x+\frac1x\right)+2=8.\]
We can expand that to get \[x^2+\frac1{x^2}-2\left(x+\frac1x\right)+2=8.\]
copeland
2017-02-16 19:40:50
Now what should we always do with an expression like this?
Now what should we always do with an expression like this?
Derive_Foiler
2017-02-16 19:41:24
x-1/x=a, and solve for the desired x^2+1/x^2
x-1/x=a, and solve for the desired x^2+1/x^2
ninjataco
2017-02-16 19:41:24
substitute a = x + 1/x
substitute a = x + 1/x
awesome_weisur
2017-02-16 19:41:24
set x + 1/x = y
set x + 1/x = y
mtarun
2017-02-16 19:41:24
Let x+1/x=a
Let x+1/x=a
First
2017-02-16 19:41:24
make $(x+\frac{1}{x}$$=y$
make $(x+\frac{1}{x}$$=y$
copeland
2017-02-16 19:41:27
We should write it as a polynomial in $x+\dfrac1x$. Since $\left(x+\dfrac1x\right)^2=x^2+\dfrac1{x^2}+2$, we write
\[\left(x+\frac1{x}\right)^2-2\left(x+\frac1x\right)=8.\]
We should write it as a polynomial in $x+\dfrac1x$. Since $\left(x+\dfrac1x\right)^2=x^2+\dfrac1{x^2}+2$, we write
\[\left(x+\frac1{x}\right)^2-2\left(x+\frac1x\right)=8.\]
copeland
2017-02-16 19:41:39
The left side is almost a square! If we add 1 we get \[\left(x+\frac1{x}-1\right)^2=9,\]and given that we're in the first quadrant we get \[\frac{x+\frac1x-1}3=1.\]
The left side is almost a square! If we add 1 we get \[\left(x+\frac1{x}-1\right)^2=9,\]and given that we're in the first quadrant we get \[\frac{x+\frac1x-1}3=1.\]
copeland
2017-02-16 19:42:09
Now what?
Now what?
mathfun5
2017-02-16 19:42:52
solve quadratic??
\
solve quadratic??
\
divijleisha
2017-02-16 19:42:52
x+1/x=4
x+1/x=4
naman12
2017-02-16 19:42:52
$x + \dfrac{1}{x} = 4$
$x + \dfrac{1}{x} = 4$
tdeng
2017-02-16 19:42:52
x+1/x=4, which we can solve as a quadratic
x+1/x=4, which we can solve as a quadratic
copeland
2017-02-16 19:42:54
We could.
We could.
copeland
2017-02-16 19:42:57
Actually it's not necessary.
Actually it's not necessary.
copeland
2017-02-16 19:43:02
Let's look more at what we have.
Let's look more at what we have.
copeland
2017-02-16 19:43:22
What's the centroid of the triangle with vertices $\left(x,\frac1x\right)$, $\left(\frac1x,x\right)$, and $\left(-1,-1\right)$?
What's the centroid of the triangle with vertices $\left(x,\frac1x\right)$, $\left(\frac1x,x\right)$, and $\left(-1,-1\right)$?
zac15SCASD
2017-02-16 19:43:49
(1, 1)
(1, 1)
warrenwangtennis
2017-02-16 19:43:49
(1, 1)
(1, 1)
donot
2017-02-16 19:43:49
1,1
1,1
Derive_Foiler
2017-02-16 19:43:49
oh. take the average of them all
oh. take the average of them all
a1b2
2017-02-16 19:43:49
$(1,1)$
$(1,1)$
HighQXMoney
2017-02-16 19:43:49
(1, 1)
(1, 1)
First
2017-02-16 19:43:49
average the x coordinates and y coordinates
average the x coordinates and y coordinates
mathislife16
2017-02-16 19:43:49
(1,1)
(1,1)
GeronimoStilton
2017-02-16 19:43:51
The average of the sum of those points?
The average of the sum of those points?
copeland
2017-02-16 19:43:53
Right!
Right!
copeland
2017-02-16 19:43:54
Now we're done. The centroid of the triangle $\left(x,\frac1x\right)$, $\left(\frac1x,x\right)$, $\left(-1,-1\right)$ is indeed \[\left(\dfrac{x+\frac1x-1}3,\dfrac{\frac1x+x-1}3\right)=(1,1).\]
Now we're done. The centroid of the triangle $\left(x,\frac1x\right)$, $\left(\frac1x,x\right)$, $\left(-1,-1\right)$ is indeed \[\left(\dfrac{x+\frac1x-1}3,\dfrac{\frac1x+x-1}3\right)=(1,1).\]
copeland
2017-02-16 19:44:00
The centroid is the circumcenter. So the medians are angle bisectors. The Angle Bisector Theorem then gives that the edge lengths are equal and we're done.
The centroid is the circumcenter. So the medians are angle bisectors. The Angle Bisector Theorem then gives that the edge lengths are equal and we're done.
copeland
2017-02-16 19:44:19
And that was a huge waste of time, testwise speaking, but at least we got to do some algebra, huh?
And that was a huge waste of time, testwise speaking, but at least we got to do some algebra, huh?
lfwei
2017-02-16 19:44:25
wow, magical
wow, magical
copeland
2017-02-16 19:44:31
It's not magic. It's math.
It's not magic. It's math.
copeland
2017-02-16 19:44:40
OK, back to business. Let's compute its area. How can we find those other two vertices?
OK, back to business. Let's compute its area. How can we find those other two vertices?
tdeng
2017-02-16 19:45:11
We don't need to
We don't need to
Derive_Foiler
2017-02-16 19:45:11
We don't need to . . .
We don't need to . . .
MSTang
2017-02-16 19:45:11
we don't need to!
we don't need to!
donot
2017-02-16 19:45:11
we don't need it
we don't need it
GeronimoStilton
2017-02-16 19:45:11
We don't need to know where they are.
We don't need to know where they are.
Derive_Foiler
2017-02-16 19:45:11
We don't
We don't
tdeng
2017-02-16 19:45:11
We don't need to; we can just find the side length!
We don't need to; we can just find the side length!
62861
2017-02-16 19:45:11
you don't need to
you don't need to
copeland
2017-02-16 19:45:15
Wait, that's a terrible idea. We can find the area much easier than that! How?
Wait, that's a terrible idea. We can find the area much easier than that! How?
EpicDragonSlayr
2017-02-16 19:46:04
side length
side length
dipenm
2017-02-16 19:46:04
The radius?
The radius?
GeronimoStilton
2017-02-16 19:46:04
We have an equilateral triangle with circumradius $2\sqrt{2}$. We can solve.
We have an equilateral triangle with circumradius $2\sqrt{2}$. We can solve.
letsgomath
2017-02-16 19:46:04
find the side length
find the side length
speulers_theorem
2017-02-16 19:46:04
distance to centroid is $2\sqrt{2}$
distance to centroid is $2\sqrt{2}$
maxplanck
2017-02-16 19:46:04
you know the circumradius
you know the circumradius
copeland
2017-02-16 19:46:06
We know the distance from the center of the equilateral triangle to one of the vertices. That determines the triangle completely. How far is it from the center to one vertex?
We know the distance from the center of the equilateral triangle to one of the vertices. That determines the triangle completely. How far is it from the center to one vertex?
copeland
2017-02-16 19:46:16
Oh, wait. You just told me.
Oh, wait. You just told me.
awesome_weisur
2017-02-16 19:46:25
2 sqrt 2
2 sqrt 2
copeland
2017-02-16 19:46:27
The distance from the center to a vertex is the distance from $(1,1)$ to $(-1,-1)$. That's $2\sqrt2$.
The distance from the center to a vertex is the distance from $(1,1)$ to $(-1,-1)$. That's $2\sqrt2$.
copeland
2017-02-16 19:46:28
So what's the altitude?
So what's the altitude?
First
2017-02-16 19:47:10
$3 \sqrt2$
$3 \sqrt2$
CornSaltButter
2017-02-16 19:47:10
3sqrt2
3sqrt2
abishek99
2017-02-16 19:47:10
3sqrt(2)
3sqrt(2)
qrohn
2017-02-16 19:47:10
3sqrt2
3sqrt2
bluequest
2017-02-16 19:47:10
3 sqrt 2
3 sqrt 2
Devo123
2017-02-16 19:47:10
3sqrt2
3sqrt2
Reef334
2017-02-16 19:47:10
3sqrt(2)
3sqrt(2)
CaptainGeo
2017-02-16 19:47:10
3sqrt2
3sqrt2
vonnan
2017-02-16 19:47:10
3root2
3root2
ImpossibleCube
2017-02-16 19:47:10
$3\sqrt{2}$
$3\sqrt{2}$
Emathmaster
2017-02-16 19:47:10
3sqrt2
3sqrt2
a000
2017-02-16 19:47:10
3sqrt2
3sqrt2
kunsun
2017-02-16 19:47:10
3sqrt2
3sqrt2
copeland
2017-02-16 19:47:12
The altitude is $\dfrac32\cdot2\sqrt2=3\sqrt2$.
The altitude is $\dfrac32\cdot2\sqrt2=3\sqrt2$.
copeland
2017-02-16 19:47:15
What's the base length?
What's the base length?
mathfun5
2017-02-16 19:47:44
2sqrt6
2sqrt6
ilovemath04
2017-02-16 19:47:44
2sqrt6
2sqrt6
Zekrom
2017-02-16 19:47:44
2*sqrt(6)
2*sqrt(6)
GeronimoStilton
2017-02-16 19:47:44
$2\sqrt{6}$
$2\sqrt{6}$
a000
2017-02-16 19:47:44
2sqrt6
2sqrt6
Devo123
2017-02-16 19:47:44
2 sqrt 6
2 sqrt 6
tree3
2017-02-16 19:47:44
2sqrt6
2sqrt6
copeland
2017-02-16 19:47:46
The length of the base is $\dfrac2{\sqrt3}\cdot 3\sqrt2=2\sqrt6$.
The length of the base is $\dfrac2{\sqrt3}\cdot 3\sqrt2=2\sqrt6$.
copeland
2017-02-16 19:47:48
So what's the square of the area?
So what's the square of the area?
ilovemath04
2017-02-16 19:48:21
108
108
abishek99
2017-02-16 19:48:21
108
108
CaptainGeo
2017-02-16 19:48:21
$\text{(C) } 108$
$\text{(C) } 108$
ImpossibleCube
2017-02-16 19:48:21
108
108
prasenjit_hazra
2017-02-16 19:48:21
108
108
copeland
2017-02-16 19:48:22
The square of the area is \[\left(\frac12\cdot3\sqrt2\cdot2\sqrt6\right)^2=\left(3\sqrt{12}\right)^2=\boxed{108}.\] The answer is (C).
The square of the area is \[\left(\frac12\cdot3\sqrt2\cdot2\sqrt6\right)^2=\left(3\sqrt{12}\right)^2=\boxed{108}.\] The answer is (C).
copeland
2017-02-16 19:48:27
Good going.
Good going.
copeland
2017-02-16 19:49:01
Should we knock the rest of this "AMC 10" out now?
Should we knock the rest of this "AMC 10" out now?
Mathaddict11
2017-02-16 19:49:16
yes!
yes!
vishwathganesan
2017-02-16 19:49:16
yeah!
yeah!
sas4
2017-02-16 19:49:16
YES
YES
PiGuy3141592
2017-02-16 19:49:16
Yes
Yes
Cardinals2014
2017-02-16 19:49:16
yeah sure
yeah sure
Mathcat1234
2017-02-16 19:49:16
sure!
sure!
divijleisha
2017-02-16 19:49:16
YES
YES
mathfun5
2017-02-16 19:49:16
yes
yes
jk23541
2017-02-16 19:49:16
yes
yes
Slacker
2017-02-16 19:49:16
last one
last one
Ani10
2017-02-16 19:49:29
why is that in quotes ;-;
why is that in quotes ;-;
Emathmaster
2017-02-16 19:49:29
lol, you put quotes
lol, you put quotes
copeland
2017-02-16 19:49:30
I love "quotes".
I love "quotes".
copeland
2017-02-16 19:50:10
25. Last year Isabella took 7 math tests and received 7 different scores, each an integer between 91 and 100, inclusive. After each test she noticed that the average of her test scores was an integer. Her score on the seventh test was 95. What was her score on the sixth test?
$\textbf{(A)}\quad 92 \qquad \qquad \textbf{(B)}\quad 94 \qquad\qquad \textbf{(C)}\quad 96 \qquad\qquad \textbf{(D)}\quad 98 \qquad\qquad\textbf{(E)}\quad 100$
25. Last year Isabella took 7 math tests and received 7 different scores, each an integer between 91 and 100, inclusive. After each test she noticed that the average of her test scores was an integer. Her score on the seventh test was 95. What was her score on the sixth test?
$\textbf{(A)}\quad 92 \qquad \qquad \textbf{(B)}\quad 94 \qquad\qquad \textbf{(C)}\quad 96 \qquad\qquad \textbf{(D)}\quad 98 \qquad\qquad\textbf{(E)}\quad 100$
copeland
2017-02-16 19:50:12
Let's set up a bunch of variables that we probably won't need. I'll call $s_1, s_2, s_3, s_4, s_5, s_6,$ and $s_7$ the scores from Isabella's seven tests.
Let's set up a bunch of variables that we probably won't need. I'll call $s_1, s_2, s_3, s_4, s_5, s_6,$ and $s_7$ the scores from Isabella's seven tests.
copeland
2017-02-16 19:50:13
Are there any other quantities we care about?
Are there any other quantities we care about?
shootingstar8
2017-02-16 19:50:29
deven!!
deven!!
copeland
2017-02-16 19:50:31
I know, right? Jerk.
I know, right? Jerk.
islander7
2017-02-16 19:51:22
their sum
their sum
naman12
2017-02-16 19:51:22
Average
Average
shootingstar8
2017-02-16 19:51:22
average
average
HighQXMoney
2017-02-16 19:51:22
Average
Average
lsh0589
2017-02-16 19:51:22
average
average
geogirl08
2017-02-16 19:51:22
averages
averages
Devo123
2017-02-16 19:51:22
the sum of the seven scores
the sum of the seven scores
MathTechFire
2017-02-16 19:51:22
average?
average?
copeland
2017-02-16 19:51:25
Averages or sums?
Averages or sums?
copeland
2017-02-16 19:51:28
Which is better?
Which is better?
mumpu2k16
2017-02-16 19:51:58
Sums.
Sums.
ninjataco
2017-02-16 19:51:58
sums
sums
mathwiz0803
2017-02-16 19:51:58
no fractions = sum
no fractions = sum
kiwitrader123
2017-02-16 19:51:58
sum
sum
alp_snow7
2017-02-16 19:51:58
Sum
Sum
mathmagician
2017-02-16 19:51:58
sums
sums
mathchampion1
2017-02-16 19:51:58
sums
sums
naman12
2017-02-16 19:51:58
Sum
Sum
Daniel2003
2017-02-16 19:51:58
sum
sum
ilovemath04
2017-02-16 19:51:58
sum
sum
quanhui868
2017-02-16 19:51:58
Sum
Sum
000libyaclawdruse000
2017-02-16 19:51:58
sums
sums
copeland
2017-02-16 19:52:00
"The average is an integer" is way worse than "The sum is divisible by. . ."
"The average is an integer" is way worse than "The sum is divisible by. . ."
copeland
2017-02-16 19:52:08
Since we're given information about the averages, we might also care about the totals, we'll let $T_1, T_2, T_3, T_4, T_5, T_6,$ and $T_7$ be Isabella's total score after each test.
Since we're given information about the averages, we might also care about the totals, we'll let $T_1, T_2, T_3, T_4, T_5, T_6,$ and $T_7$ be Isabella's total score after each test.
copeland
2017-02-16 19:52:10
Since they gave us information about the seventh test, it makes sense to start there. What do we know?
Since they gave us information about the seventh test, it makes sense to start there. What do we know?
SomethingNeutral
2017-02-16 19:52:34
s7 = 95
s7 = 95
pie314159265
2017-02-16 19:52:34
the score was 95
the score was 95
a1b2
2017-02-16 19:52:34
$s_7=95$
$s_7=95$
awesomemaths
2017-02-16 19:52:34
the sevnth test score is a 95
the sevnth test score is a 95
copeland
2017-02-16 19:52:37
We know she scored a 95 on the seventh test. So, $s_7 = 95.$
We know she scored a 95 on the seventh test. So, $s_7 = 95.$
copeland
2017-02-16 19:52:40
Additionally, we know that $T_7$ is a multiple of 7.
Additionally, we know that $T_7$ is a multiple of 7.
copeland
2017-02-16 19:52:43
There are lots of multiples of 7... can we narrow it down at all?
There are lots of multiples of 7... can we narrow it down at all?
islander7
2017-02-16 19:54:09
between 7*94 and 7*97
between 7*94 and 7*97
Fridaychimp
2017-02-16 19:54:09
94*7-97*7
94*7-97*7
SomethingNeutral
2017-02-16 19:54:09
91+...97 to 94+..100
91+...97 to 94+..100
Ani10
2017-02-16 19:54:09
91+92+93+94+95+96+97 to 94+95+96+97+98+99+100 multiples of 7 inbetween
91+92+93+94+95+96+97 to 94+95+96+97+98+99+100 multiples of 7 inbetween
mathislife16
2017-02-16 19:54:09
94*7 through 97*7
94*7 through 97*7
copeland
2017-02-16 19:54:12
We know the lowest she could have possibly scored is $91 + 92 + 93 + 94 + 95 + 96 + 97 = 658.$
We know the lowest she could have possibly scored is $91 + 92 + 93 + 94 + 95 + 96 + 97 = 658.$
copeland
2017-02-16 19:54:14
The highest she could have possibly scored is $100 + 99 + 98 + 97 +96 + 95 + 94 = 679.$
The highest she could have possibly scored is $100 + 99 + 98 + 97 +96 + 95 + 94 = 679.$
copeland
2017-02-16 19:54:14
What are the multiples of 7 in that range?
What are the multiples of 7 in that range?
kunsun
2017-02-16 19:54:55
658 665 672 679
658 665 672 679
vishwathganesan
2017-02-16 19:54:55
658, 665, 672, 679
658, 665, 672, 679
weilunsun28
2017-02-16 19:54:55
658 665 672 679
658 665 672 679
mathwrite
2017-02-16 19:54:55
658, 665, 672, 679
658, 665, 672, 679
letsgomath
2017-02-16 19:54:55
658, 665, 672, 679
658, 665, 672, 679
SomethingNeutral
2017-02-16 19:54:55
658, 665, 672, 679
658, 665, 672, 679
azmath333
2017-02-16 19:54:55
658, 665, 672, 679
658, 665, 672, 679
copeland
2017-02-16 19:54:57
We know 658 and 679 are multiples of 7 since they're the sum of 7 consecutive integers, so we just list the others in between. Our possibilities are 658, 665, 672, and 679.
We know 658 and 679 are multiples of 7 since they're the sum of 7 consecutive integers, so we just list the others in between. Our possibilities are 658, 665, 672, and 679.
copeland
2017-02-16 19:55:04
Now we're in business. How can we narrow that down even further?
Now we're in business. How can we narrow that down even further?
First
2017-02-16 19:55:48
Remove 95 and see if it's divisble by 6
Remove 95 and see if it's divisble by 6
Derive_Foiler
2017-02-16 19:55:48
subtract 95 gives a multiple of 6
subtract 95 gives a multiple of 6
islander7
2017-02-16 19:55:48
-95 becomes a multiple of 6
-95 becomes a multiple of 6
GeronimoStilton
2017-02-16 19:55:48
If we subtract $95$, we will get a multiple of $6$
If we subtract $95$, we will get a multiple of $6$
SomethingNeutral
2017-02-16 19:55:48
-95 is multiple of 6
-95 is multiple of 6
mathmagician
2017-02-16 19:55:48
the possibilities - 95 is divisible by 6
the possibilities - 95 is divisible by 6
lsh0589
2017-02-16 19:55:48
-95 and see if that's a multiple of 6
-95 and see if that's a multiple of 6
mathwizard666
2017-02-16 19:55:48
95 less than the number is a multiple of 6
95 less than the number is a multiple of 6
carfan25
2017-02-16 19:55:48
take away 95, the result has to be a multiple of 6
take away 95, the result has to be a multiple of 6
DaAsianPotatoe
2017-02-16 19:55:50
T_7 - 95 = 6*x
T_7 - 95 = 6*x
copeland
2017-02-16 19:55:54
We know that $T_6 = T_7 - 95$ is a multiple of 6. Do any of these satisfy that?
We know that $T_6 = T_7 - 95$ is a multiple of 6. Do any of these satisfy that?
SomethingNeutral
2017-02-16 19:56:42
665.
665.
tnalluri
2017-02-16 19:56:42
665
665
mathfun5
2017-02-16 19:56:42
665
665
Reef334
2017-02-16 19:56:42
665
665
tdeng
2017-02-16 19:56:42
665
665
mathfun5
2017-02-16 19:56:42
only 665
only 665
Cardinals2014
2017-02-16 19:56:42
665
665
abishek99
2017-02-16 19:56:42
only 665
only 665
divijleisha
2017-02-16 19:56:42
665
665
AAANNNMMMIII
2017-02-16 19:56:42
665
665
Slacker
2017-02-16 19:56:42
665
665
AOPS12142015
2017-02-16 19:56:42
665 works
665 works
Happytycho
2017-02-16 19:56:42
665
665
copeland
2017-02-16 19:56:43
Only $665 - 95 = 570$ is a multiple of 6. So, $T_7 = 665$ and $T_6 = 570.$
Only $665 - 95 = 570$ is a multiple of 6. So, $T_7 = 665$ and $T_6 = 570.$
copeland
2017-02-16 19:56:46
Alright. Let's just keep working backwards. We know $T_5$ is a multiple of 5. How does $T_5$ relate to $T_6$?
Alright. Let's just keep working backwards. We know $T_5$ is a multiple of 5. How does $T_5$ relate to $T_6$?
vishwathganesan
2017-02-16 19:57:39
the difference is the answer
the difference is the answer
jeffshen
2017-02-16 19:57:39
100
100
tdeng
2017-02-16 19:57:39
They are both multiples of 5.
They are both multiples of 5.
Piazolla13
2017-02-16 19:57:39
T6 - s6
T6 - s6
hinna
2017-02-16 19:57:39
$T_5+s_6=T_6$
$T_5+s_6=T_6$
SomethingNeutral
2017-02-16 19:57:39
t6-t5 is a multiple of 5.
t6-t5 is a multiple of 5.
Reef334
2017-02-16 19:57:39
T_6 - S_6 = T_5
T_6 - S_6 = T_5
Slacker
2017-02-16 19:57:39
$T_5+s_6=T_6$
$T_5+s_6=T_6$
copeland
2017-02-16 19:57:42
We know $T_5 = T_6 - s_6$. Does that tell us anything about $s_6$?
We know $T_5 = T_6 - s_6$. Does that tell us anything about $s_6$?
ninjataco
2017-02-16 19:58:21
s6 is a mulitple of 5
s6 is a mulitple of 5
belieber
2017-02-16 19:58:21
divisible by 5
divisible by 5
shootingstar8
2017-02-16 19:58:21
Is a multiple of 5
Is a multiple of 5
Zekrom
2017-02-16 19:58:21
s_6 is a multiple of 5
s_6 is a multiple of 5
Devo123
2017-02-16 19:58:21
multiple of 5
multiple of 5
Antidifferentiation108
2017-02-16 19:58:21
is multiple of 5
is multiple of 5
AlcumusGuy
2017-02-16 19:58:21
needs to be multiple of 5
needs to be multiple of 5
Smoothfang
2017-02-16 19:58:21
Multiple of 5
Multiple of 5
copeland
2017-02-16 19:58:23
Since $T_6$ is a multiple of 5, we know that $s_6$ must also be a multiple of 5. So?
Since $T_6$ is a multiple of 5, we know that $s_6$ must also be a multiple of 5. So?
SomethingNeutral
2017-02-16 19:58:49
s6 = 95 or 100 but 95 is taken
s6 = 95 or 100 but 95 is taken
tdeng
2017-02-16 19:58:49
Must be 0(mod5), 95 is taken, so it must be 100
Must be 0(mod5), 95 is taken, so it must be 100
Piazolla13
2017-02-16 19:58:49
multiple of 5 so 100
multiple of 5 so 100
jfmath04
2017-02-16 19:58:49
it has to be 100 in order for T_5 to be a multiple of 5
it has to be 100 in order for T_5 to be a multiple of 5
islander7
2017-02-16 19:58:49
s6=100
s6=100
letsgomath
2017-02-16 19:58:49
not 95 so E 100
not 95 so E 100
kiwitrader123
2017-02-16 19:58:49
only 100 is multipel
only 100 is multipel
dilworthpenguins
2017-02-16 19:58:49
it must be 100 since it can't be 95
it must be 100 since it can't be 95
Cidkip
2017-02-16 19:58:49
100 is a multiple of 5, none of the other answer choices are
100 is a multiple of 5, none of the other answer choices are
rspr2001
2017-02-16 19:58:49
The answer is E!
The answer is E!
ilovemath04
2017-02-16 19:58:49
s6 must be 100
s6 must be 100
summitwei
2017-02-16 19:58:49
it has to be 100
it has to be 100
mathmagician
2017-02-16 19:58:49
s6 is 100, since 95 is s7
s6 is 100, since 95 is s7
copeland
2017-02-16 19:58:51
The only multiple of 5 available is 100. We conclude that $s_6 = \boxed{100}.$
The only multiple of 5 available is 100. We conclude that $s_6 = \boxed{100}.$
copeland
2017-02-16 19:58:52
Our answer is (E).
Our answer is (E).
copeland
2017-02-16 19:58:58
I think there are 4 sequences that work.
I think there are 4 sequences that work.
copeland
2017-02-16 19:59:00
$92+96+91+97+94+100+95.$
$92+96+91+97+94+100+95.$
copeland
2017-02-16 19:59:00
$91+93+92+96+98+100+95$
$91+93+92+96+98+100+95$
copeland
2017-02-16 19:59:05
And swap the first 2 elements of each of those.
And swap the first 2 elements of each of those.
mumpu2k16
2017-02-16 19:59:30
AMC 12 now?
AMC 12 now?
copeland
2017-02-16 19:59:31
Eh. What do you think?
Eh. What do you think?
Derive_Foiler
2017-02-16 19:59:58
yea!
yea!
awesomemaths
2017-02-16 19:59:58
YEAH
YEAH
JP-GoVikes
2017-02-16 19:59:58
yessss
yessss
techguy2
2017-02-16 19:59:58
10 better
10 better
First
2017-02-16 19:59:58
Sure
Sure
mumpu2k16
2017-02-16 19:59:58
Of course!
Of course!
SaltySnail2
2017-02-16 19:59:58
i wish to do amc 12, if possible
i wish to do amc 12, if possible
speulers_theorem
2017-02-16 19:59:58
Yeah!
Yeah!
MathTechFire
2017-02-16 19:59:58
Fine
Fine
copeland
2017-02-16 20:00:09
That's the kind of mostly-unfettered enthusiasm I love.
That's the kind of mostly-unfettered enthusiasm I love.
copeland
2017-02-16 20:00:20
But we won't start with Problem 21.
But we won't start with Problem 21.
copeland
2017-02-16 20:00:24
AMC 10B Problem 25 and AMC 12B Problem 21 were the same. Let's move on to Problem 22.
AMC 10B Problem 25 and AMC 12B Problem 21 were the same. Let's move on to Problem 22.
copeland
2017-02-16 20:00:42
(I found it weird that 25 got demoted to 21 when it crossed over. Usually it lands later.)
(I found it weird that 25 got demoted to 21 when it crossed over. Usually it lands later.)
copeland
2017-02-16 20:00:48
22. Abby, Bernardo, Carl, and Debra play a game in which each of them starts with four coins. The game consists of four rounds. In each round, four balls are placed in an urn---one green, one red, and two white. The players each draw a ball at random without replacement. Whoever gets the green ball gives one coin to whoever gets the red ball. What is the probability that, at the end of the fourth round, each of the players has four coins?
$\textbf{(A)}\quad \dfrac{7}{576} \qquad \qquad \textbf{(B)}\quad \dfrac{5}{192} \qquad\qquad \textbf{(C)}\quad \dfrac{1}{36} \qquad\qquad \textbf{(D)}\quad \dfrac{5}{144} \qquad\qquad\textbf{(E)}\quad \dfrac{7}{48}$
22. Abby, Bernardo, Carl, and Debra play a game in which each of them starts with four coins. The game consists of four rounds. In each round, four balls are placed in an urn---one green, one red, and two white. The players each draw a ball at random without replacement. Whoever gets the green ball gives one coin to whoever gets the red ball. What is the probability that, at the end of the fourth round, each of the players has four coins?
$\textbf{(A)}\quad \dfrac{7}{576} \qquad \qquad \textbf{(B)}\quad \dfrac{5}{192} \qquad\qquad \textbf{(C)}\quad \dfrac{1}{36} \qquad\qquad \textbf{(D)}\quad \dfrac{5}{144} \qquad\qquad\textbf{(E)}\quad \dfrac{7}{48}$
copeland
2017-02-16 20:01:00
This sounds like a boring game.
This sounds like a boring game.
Slacker
2017-02-16 20:01:09
ikr
ikr
letsgomath
2017-02-16 20:01:12
i know right
i know right
copeland
2017-02-16 20:01:14
OK, how many total ways are there for the 4 players to draw these balls?
OK, how many total ways are there for the 4 players to draw these balls?
Ani10
2017-02-16 20:01:30
i got this problem right by playing it 400 times
i got this problem right by playing it 400 times
copeland
2017-02-16 20:01:31
Me too!
Me too!
vishwathganesan
2017-02-16 20:02:15
4!/2 = 12
4!/2 = 12
speulers_theorem
2017-02-16 20:02:15
12
12
yrnsmurf
2017-02-16 20:02:15
12
12
Superwiz
2017-02-16 20:02:15
12
12
GeronimoStilton
2017-02-16 20:02:15
$12$
$12$
rspr2001
2017-02-16 20:02:15
12
12
copeland
2017-02-16 20:02:20
On each turn, there are 4 ways to hand out the green ball then three ways to hand out the red one. That's $4\cdot3=12$ ways to draw each turn.
On each turn, there are 4 ways to hand out the green ball then three ways to hand out the red one. That's $4\cdot3=12$ ways to draw each turn.
copeland
2017-02-16 20:02:30
Four turns gives $12^4$. Great. That's our denominator.
Four turns gives $12^4$. Great. That's our denominator.
copeland
2017-02-16 20:02:33
Now how can we work out the numerator?
Now how can we work out the numerator?
ilovemath04
2017-02-16 20:03:18
casework
casework
SomethingNeutral
2017-02-16 20:03:18
brute force or casework
brute force or casework
yrnsmurf
2017-02-16 20:03:18
casework and graph theory?
casework and graph theory?
Smoothfang
2017-02-16 20:03:18
Casework?
Casework?
summitwei
2017-02-16 20:03:18
casework
casework
copeland
2017-02-16 20:03:20
I guess we use casework. We haven't seen any casework yet and it is the AMC.
I guess we use casework. We haven't seen any casework yet and it is the AMC.
copeland
2017-02-16 20:03:21
How should we organize the cases?
How should we organize the cases?
copeland
2017-02-16 20:03:24
First, how many times can a person draw the green ball?
First, how many times can a person draw the green ball?
Derive_Foiler
2017-02-16 20:04:06
they can draw it once or twice
they can draw it once or twice
mathislife16
2017-02-16 20:04:06
twice or it will fail
twice or it will fail
Superwiz
2017-02-16 20:04:06
0,1or 2 times
0,1or 2 times
GeronimoStilton
2017-02-16 20:04:06
$2$ for the event to happen
$2$ for the event to happen
geogirl08
2017-02-16 20:04:06
2, 1, or 0
2, 1, or 0
copeland
2017-02-16 20:04:10
A person can draw the green ball twice if he draws the red ball the other 2 times.
A person can draw the green ball twice if he draws the red ball the other 2 times.
copeland
2017-02-16 20:04:18
He could draw it less, of course.
He could draw it less, of course.
copeland
2017-02-16 20:04:21
Let's first consider the cases where someone draws the green ball twice.
Let's first consider the cases where someone draws the green ball twice.
copeland
2017-02-16 20:04:28
Say Abby draws the green ball twice and the red ball twice. That means she gets two coins from other players. Either those players are the same or different:
Say Abby draws the green ball twice and the red ball twice. That means she gets two coins from other players. Either those players are the same or different:
copeland
2017-02-16 20:04:33
copeland
2017-02-16 20:04:33
copeland
2017-02-16 20:04:40
Look how big those diagrams are!
Look how big those diagrams are!
copeland
2017-02-16 20:04:44
You know who's fault that is?
You know who's fault that is?
Reef334
2017-02-16 20:04:57
deven
deven
shakeNbake
2017-02-16 20:04:57
deven
deven
tdeng
2017-02-16 20:04:57
Deven!
Deven!
SomethingNeutral
2017-02-16 20:04:57
deven
deven
kiwitrader123
2017-02-16 20:04:57
DEVEN
DEVEN
naman12
2017-02-16 20:04:57
devenware
devenware
000libyaclawdruse000
2017-02-16 20:04:57
Deven!
Deven!
copeland
2017-02-16 20:05:00
Right.
Right.
copeland
2017-02-16 20:05:04
If Abby draws two greens and passes to Bernardo both times then what has to happen on the other two draws?
If Abby draws two greens and passes to Bernardo both times then what has to happen on the other two draws?
tdeng
2017-02-16 20:06:10
Bernardo has to pass to Abby both times.
Bernardo has to pass to Abby both times.
vishwathganesan
2017-02-16 20:06:10
bernardo draws both greens next
bernardo draws both greens next
DaAsianPotatoe
2017-02-16 20:06:10
Bernardo passes them back
Bernardo passes them back
geogirl08
2017-02-16 20:06:10
abby draws red twice and bernardo draws green twice
abby draws red twice and bernardo draws green twice
oumarrbah_clf
2017-02-16 20:06:10
bernando has to draw two greens and abby has to draw two reds
bernando has to draw two greens and abby has to draw two reds
mathislife16
2017-02-16 20:06:10
B passes to A
B passes to A
Happytycho
2017-02-16 20:06:10
pass them back
pass them back
summitwei
2017-02-16 20:06:10
Bernardo passes to Abby twice
Bernardo passes to Abby twice
islander7
2017-02-16 20:06:10
bernardo passes to abby other 2 times
bernardo passes to abby other 2 times
copeland
2017-02-16 20:06:12
If Abby passes twice to Bernard, then Bernardo must also pass twice to Abby.
If Abby passes twice to Bernard, then Bernardo must also pass twice to Abby.
copeland
2017-02-16 20:06:13
copeland
2017-02-16 20:06:19
Now THAT is a diagram.
Now THAT is a diagram.
copeland
2017-02-16 20:06:24
How many total different draws have two people exchanging coins in this way? (Any two people, not just Abby and Bernardo.)
How many total different draws have two people exchanging coins in this way? (Any two people, not just Abby and Bernardo.)
sas4
2017-02-16 20:07:16
36
36
ilovemath04
2017-02-16 20:07:16
36
36
vishwathganesan
2017-02-16 20:07:16
4 choose 2 * 4 choose 2 = 36?
4 choose 2 * 4 choose 2 = 36?
SomethingNeutral
2017-02-16 20:07:16
4 choose 2 times 4 choose 2 equals 36.
4 choose 2 times 4 choose 2 equals 36.
mathfun5
2017-02-16 20:07:16
36
36
Superwiz
2017-02-16 20:07:16
6* 6=36
6* 6=36
mathfun5
2017-02-16 20:07:16
6*6
6*6
Tribefan
2017-02-16 20:07:16
4C2*4C2 = 36
4C2*4C2 = 36
geogirl08
2017-02-16 20:07:16
6*6
6*6
copeland
2017-02-16 20:07:18
There are 6 ways to choose a pair of people. Once we choose them, there are $\binom42=6$ ways to choose which turns the coins move one way and which turns the coins move the other. There are 36 total draws that have this form.
There are 6 ways to choose a pair of people. Once we choose them, there are $\binom42=6$ ways to choose which turns the coins move one way and which turns the coins move the other. There are 36 total draws that have this form.
copeland
2017-02-16 20:07:34
What happens if Abby draws two greens and passes once to Bernardo and once to Carl? What can happen on the other two turns?
What happens if Abby draws two greens and passes once to Bernardo and once to Carl? What can happen on the other two turns?
Superwiz
2017-02-16 20:08:16
C to A, B to A
C to A, B to A
GeronimoStilton
2017-02-16 20:08:16
Bernardo and Carl pass back.
Bernardo and Carl pass back.
tdeng
2017-02-16 20:08:16
Bernardo has to pass to Abby and Carl has to pass to Abby
Bernardo has to pass to Abby and Carl has to pass to Abby
First
2017-02-16 20:08:16
They each give a coin to Abby
They each give a coin to Abby
vishwathganesan
2017-02-16 20:08:16
Abby draws 2 reds and carl and barnardo each draw a green
Abby draws 2 reds and carl and barnardo each draw a green
yrnsmurf
2017-02-16 20:08:16
C to A and B to A
C to A and B to A
ninjataco
2017-02-16 20:08:16
bernardo and carl pass back
bernardo and carl pass back
oumarrbah_clf
2017-02-16 20:08:16
Abby has to draw two reds and bernando and carl each have to draw one green
Abby has to draw two reds and bernando and carl each have to draw one green
copeland
2017-02-16 20:08:20
copeland
2017-02-16 20:08:20
On the other two turns, Bernardo and Carl have to pass once each and Abby needs to receive two coins. Therefore Bernardo and Carl must both pass to Abby on the other two turns.
On the other two turns, Bernardo and Carl have to pass once each and Abby needs to receive two coins. Therefore Bernardo and Carl must both pass to Abby on the other two turns.
copeland
2017-02-16 20:08:23
copeland
2017-02-16 20:08:27
How many total different draws have this L-shape? (Among any set of 3 players.)
How many total different draws have this L-shape? (Among any set of 3 players.)
mathfun5
2017-02-16 20:09:45
12*24 = 288
12*24 = 288
linqaszayi
2017-02-16 20:09:45
4*3*4*3*2=288
4*3*4*3*2=288
mathfun5
2017-02-16 20:09:45
288
288
GeronimoStilton
2017-02-16 20:09:45
4 ways to choose the first person times three ways to choose the person who doesn't participate times 24 ways to choose the order = 288$
4 ways to choose the first person times three ways to choose the person who doesn't participate times 24 ways to choose the order = 288$
copeland
2017-02-16 20:09:48
There are 4 people to choose as the node of the $L$ and then 3 choices for the lonely player. Then all four edges are distinct so there are $4!=24$ orders in which to perform the swaps. That gives all of $4\cdot3\cdot24=288$ ways for this type of swap to happen.
There are 4 people to choose as the node of the $L$ and then 3 choices for the lonely player. Then all four edges are distinct so there are $4!=24$ orders in which to perform the swaps. That gives all of $4\cdot3\cdot24=288$ ways for this type of swap to happen.
copeland
2017-02-16 20:09:50
OK, otherwise nobody draws two greens. Then what happens?
OK, otherwise nobody draws two greens. Then what happens?
ninjataco
2017-02-16 20:10:54
everyone draws 1
everyone draws 1
vishwathganesan
2017-02-16 20:10:54
each person draws one green and one red
each person draws one green and one red
letsgomath
2017-02-16 20:10:54
they all draw one?
they all draw one?
GeronimoStilton
2017-02-16 20:10:54
Everybody draws a green.
Everybody draws a green.
First
2017-02-16 20:10:54
Everyone draws a green
Everyone draws a green
ilovemath04
2017-02-16 20:10:54
everyone draws one green
everyone draws one green
duck_master
2017-02-16 20:10:54
one green one red two white
one green one red two white
copeland
2017-02-16 20:11:08
Since there are 4 greens drawn, everyone draws one green. Likewise for red. What are all the configurations where each person draws a single green and a single red?
Since there are 4 greens drawn, everyone draws one green. Likewise for red. What are all the configurations where each person draws a single green and a single red?
copeland
2017-02-16 20:11:55
This is a question about graphs, not a question about numbers.
This is a question about graphs, not a question about numbers.
copeland
2017-02-16 20:12:10
Say Abby passes to Bernardo again. If Bernardo passes back to Abby then what?
Say Abby passes to Bernardo again. If Bernardo passes back to Abby then what?
copeland
2017-02-16 20:12:12
Derive_Foiler
2017-02-16 20:12:38
CD do a swap
CD do a swap
geogirl08
2017-02-16 20:12:38
Carl passes to Debra and back
Carl passes to Debra and back
mathislife16
2017-02-16 20:12:38
the other two do a swap
the other two do a swap
yrnsmurf
2017-02-16 20:12:38
C and D have to trade
C and D have to trade
Reef334
2017-02-16 20:12:38
C and D pass to each other
C and D pass to each other
copeland
2017-02-16 20:12:41
That means Carl and Debra have to swap as well:
That means Carl and Debra have to swap as well:
copeland
2017-02-16 20:12:42
copeland
2017-02-16 20:12:49
Therefore we get a pair of swaps.
Therefore we get a pair of swaps.
copeland
2017-02-16 20:12:53
However, if Abby passes to Bernardo and Bernardo then passes to Debra:
However, if Abby passes to Bernardo and Bernardo then passes to Debra:
copeland
2017-02-16 20:12:54
copeland
2017-02-16 20:12:56
Then what?
Then what?
SomethingNeutral
2017-02-16 20:13:34
cycle 4
cycle 4
vishwathganesan
2017-02-16 20:13:34
debra to carl and carl to abby
debra to carl and carl to abby
mathislife16
2017-02-16 20:13:34
D to C and C to A
D to C and C to A
Reef334
2017-02-16 20:13:34
D passes to C then to A
D passes to C then to A
Derive_Foiler
2017-02-16 20:13:34
D to C, C to A?
D to C, C to A?
Math-101
2017-02-16 20:13:34
A circle
A circle
GeronimoStilton
2017-02-16 20:13:34
Debra passes to Carl and Carl passes to Abby
Debra passes to Carl and Carl passes to Abby
MathTechFire
2017-02-16 20:13:34
A square cycle
A square cycle
Rushn
2017-02-16 20:13:34
D passes to C who passes to A
D passes to C who passes to A
summitwei
2017-02-16 20:13:34
D->C and C->A
D->C and C->A
abishek99
2017-02-16 20:13:34
go around the square
go around the square
copeland
2017-02-16 20:13:37
Then Bernardo is done playing. Now Carl still needs to pass to someone but the only one left to receive is Abby. So Carl passes to Abby and Debra must pass to Carl.
Then Bernardo is done playing. Now Carl still needs to pass to someone but the only one left to receive is Abby. So Carl passes to Abby and Debra must pass to Carl.
copeland
2017-02-16 20:13:40
copeland
2017-02-16 20:13:41
So, we have to have either a cycle or a pair of swaps.
So, we have to have either a cycle or a pair of swaps.
copeland
2017-02-16 20:13:43
How many draws produce this type of configuration?
How many draws produce this type of configuration?
copeland
2017-02-16 20:13:43
linqaszayi
2017-02-16 20:14:54
3*4!=72
3*4!=72
geogirl08
2017-02-16 20:14:54
3*24
3*24
vishwathganesan
2017-02-16 20:14:54
3*24 = 72
3*24 = 72
summitwei
2017-02-16 20:14:54
3*24
3*24
copeland
2017-02-16 20:14:57
There are 3 ways to pair off the players (pick Abby's partner). Once that's decided, the draws are all distinct so there are $4!=24$ ways to order the draws. There are $3\cdot24=72$ sets of draws in this case.
There are 3 ways to pair off the players (pick Abby's partner). Once that's decided, the draws are all distinct so there are $4!=24$ ways to order the draws. There are $3\cdot24=72$ sets of draws in this case.
copeland
2017-02-16 20:14:58
What about if it's a cycle? How many ways can the players draw a cycle?
What about if it's a cycle? How many ways can the players draw a cycle?
copeland
2017-02-16 20:14:59
summitwei
2017-02-16 20:16:22
6*24
6*24
linqaszayi
2017-02-16 20:16:22
3*2*4!=144
3*2*4!=144
vishwathganesan
2017-02-16 20:16:22
no 144!
no 144!
islander7
2017-02-16 20:16:22
6*4!
6*4!
tdeng
2017-02-16 20:16:22
144
144
copeland
2017-02-16 20:16:24
There are 3 choices for who Abby passes to and then 2 choices for who passes to Abby. That gives 6 total ways to form a cycle. There are again $4!=24$ orders to draw the cycle, so there are $6\cdot24=144$ total sets draws in this case.
There are 3 choices for who Abby passes to and then 2 choices for who passes to Abby. That gives 6 total ways to form a cycle. There are again $4!=24$ orders to draw the cycle, so there are $6\cdot24=144$ total sets draws in this case.
copeland
2017-02-16 20:16:25
What's the final answer?
What's the final answer?
duck_master
2017-02-16 20:18:47
540/12^4
540/12^4
Tribefan
2017-02-16 20:18:47
540/(12^4) = 5/192 (B)
540/(12^4) = 5/192 (B)
duck_master
2017-02-16 20:18:47
B) 5/192
B) 5/192
abishek99
2017-02-16 20:18:47
5/192 --> B
5/192 --> B
GeronimoStilton
2017-02-16 20:18:47
B
B
rt03
2017-02-16 20:18:47
5/192
5/192
copeland
2017-02-16 20:18:49
The total probability is
\begin{align*}
P
&=\frac{36+288+72+144}{12^4}\\
&=\frac{3+24+6+12}{12^3}\\
&=\frac{45}{12^3}\\
&=\frac{5}{4\cdot4\cdot12}\\
&=\boxed{\dfrac{5}{192}}.
\end{align*}
The total probability is
\begin{align*}
P
&=\frac{36+288+72+144}{12^4}\\
&=\frac{3+24+6+12}{12^3}\\
&=\frac{45}{12^3}\\
&=\frac{5}{4\cdot4\cdot12}\\
&=\boxed{\dfrac{5}{192}}.
\end{align*}
copeland
2017-02-16 20:18:50
The answer is (B).
The answer is (B).
copeland
2017-02-16 20:18:54
Ready to move on?
Ready to move on?
copeland
2017-02-16 20:19:05
I sense that was not everyone's favorite problem.
I sense that was not everyone's favorite problem.
tfz4629
2017-02-16 20:19:11
Sure
Sure
mathchampion1
2017-02-16 20:19:11
yes!
yes!
HighQXMoney
2017-02-16 20:19:11
YES
YES
HighQXMoney
2017-02-16 20:19:11
YES
YES
YES
YES
YES
YES
copeland
2017-02-16 20:19:13
Great.
Great.
copeland
2017-02-16 20:19:27
I "solved" that problem at least a dozen times before I got the right answer.
I "solved" that problem at least a dozen times before I got the right answer.
copeland
2017-02-16 20:19:36
23. The graph of $y=f(x)$, where $f(x)$ is a polynomial of degree 3, contains points $A(2,4)$, $B(3,9)$, and $C(4,16)$. Lines $AB$, $AC$, and $BC$ intersect the graph again at points $D$, $E$, and $F$, respectively, and the sum of the $x$-coordinates of $D$, $E$, and $F$ is 24. What is $f(0)$?
$\textbf{(A)}\quad {-2} \qquad \qquad \textbf{(B)}\quad 0 \qquad\qquad \textbf{(C)}\quad 2 \qquad\qquad \textbf{(D)}\quad \dfrac{24}5 \qquad\qquad\textbf{(E)}\quad 8$
23. The graph of $y=f(x)$, where $f(x)$ is a polynomial of degree 3, contains points $A(2,4)$, $B(3,9)$, and $C(4,16)$. Lines $AB$, $AC$, and $BC$ intersect the graph again at points $D$, $E$, and $F$, respectively, and the sum of the $x$-coordinates of $D$, $E$, and $F$ is 24. What is $f(0)$?
$\textbf{(A)}\quad {-2} \qquad \qquad \textbf{(B)}\quad 0 \qquad\qquad \textbf{(C)}\quad 2 \qquad\qquad \textbf{(D)}\quad \dfrac{24}5 \qquad\qquad\textbf{(E)}\quad 8$
copeland
2017-02-16 20:19:39
OK, so what do you notice about those points?
OK, so what do you notice about those points?
ninjataco
2017-02-16 20:20:20
satisfy y = x^2
satisfy y = x^2
Derive_Foiler
2017-02-16 20:20:20
hmm . . . (x, x^2)
hmm . . . (x, x^2)
Reef334
2017-02-16 20:20:20
(x, x^2)
(x, x^2)
Tribefan
2017-02-16 20:20:20
Of the form (x,x^2)
Of the form (x,x^2)
SmartGuy101
2017-02-16 20:20:20
Squares
Squares
JP-GoVikes
2017-02-16 20:20:20
squares
squares
SKundu13
2017-02-16 20:20:20
y = x^2
y = x^2
pie314159265
2017-02-16 20:20:20
they all lie on y=x^2
they all lie on y=x^2
DaAsianPotatoe
2017-02-16 20:20:20
y = x^2
y = x^2
SmartGuy101
2017-02-16 20:20:20
y = x^2
y = x^2
copeland
2017-02-16 20:20:22
They're all of the form $(x,x^2)$.
They're all of the form $(x,x^2)$.
copeland
2017-02-16 20:20:24
So how should we "encode" that in $f$?
So how should we "encode" that in $f$?
acegikmoqsuwy2000
2017-02-16 20:21:04
$f(x)-x^2$ equals zero at those three values (and hence has them as roots)
$f(x)-x^2$ equals zero at those three values (and hence has them as roots)
yrnsmurf
2017-02-16 20:21:04
f(x)-x^2 has roots 2,3,4
f(x)-x^2 has roots 2,3,4
rt03
2017-02-16 20:21:04
add a cubic polynomial with roots 2,3,4 to x^2
add a cubic polynomial with roots 2,3,4 to x^2
copeland
2017-02-16 20:21:07
What if we write $f(x)=x^2+\text{something}$ What do we know about the something?
What if we write $f(x)=x^2+\text{something}$ What do we know about the something?
ninjataco
2017-02-16 20:21:54
f(x) - x^2 = a(x-2)(x-3)(x-4)
f(x) - x^2 = a(x-2)(x-3)(x-4)
pie314159265
2017-02-16 20:21:54
has roots of 2, 3, 4
has roots of 2, 3, 4
Derive_Foiler
2017-02-16 20:21:54
it has zeros at 2, 3, 4
it has zeros at 2, 3, 4
GeronimoStilton
2017-02-16 20:21:54
$f(x) = x^2 + a(x-2)(x-3)(x-4)$
$f(x) = x^2 + a(x-2)(x-3)(x-4)$
abishek99
2017-02-16 20:21:54
0 for x = 2,3,4
0 for x = 2,3,4
Derive_Foiler
2017-02-16 20:21:54
it's a(x-2)(x-3)(x-4)
it's a(x-2)(x-3)(x-4)
ilovemath04
2017-02-16 20:21:54
has roots 2,3, and 4
has roots 2,3, and 4
copeland
2017-02-16 20:21:56
The something has roots at 2, 3, and 4. It is also a cubic polynomial, so it is of the form $a(x-2)(x-3)(x-4)$.
The something has roots at 2, 3, and 4. It is also a cubic polynomial, so it is of the form $a(x-2)(x-3)(x-4)$.
copeland
2017-02-16 20:21:58
\[f(x)=x^2+a(x-2)(x-3)(x-4).\]
\[f(x)=x^2+a(x-2)(x-3)(x-4).\]
copeland
2017-02-16 20:22:00
OK, so about the line through $AB$? What line is that?
OK, so about the line through $AB$? What line is that?
flyrain
2017-02-16 20:22:59
5x-6
5x-6
Derive_Foiler
2017-02-16 20:22:59
that's y=5x-6
that's y=5x-6
Tribefan
2017-02-16 20:22:59
5x-6
5x-6
vishwathganesan
2017-02-16 20:22:59
y=5x-6
y=5x-6
Reef334
2017-02-16 20:22:59
y=5x-6
y=5x-6
HighQXMoney
2017-02-16 20:22:59
Line is $y=5x-6$
Line is $y=5x-6$
shakeNbake
2017-02-16 20:22:59
y=5x-6
y=5x-6
thedoge
2017-02-16 20:22:59
$y = 5x-6$
$y = 5x-6$
copeland
2017-02-16 20:23:01
That line has slope $\dfrac{9-4}{3-2}=5$, so it is $y=5x-6$.
That line has slope $\dfrac{9-4}{3-2}=5$, so it is $y=5x-6$.
copeland
2017-02-16 20:23:04
How do we find the $x$-value of the other root?
How do we find the $x$-value of the other root?
geogirl08
2017-02-16 20:24:01
=
=
copeland
2017-02-16 20:24:08
One brilliant character. What's it mean?
One brilliant character. What's it mean?
geogirl08
2017-02-16 20:25:02
set them equal
set them equal
brainiac1
2017-02-16 20:25:02
equate the expressions
equate the expressions
Derive_Foiler
2017-02-16 20:25:02
set them equal?
set them equal?
islander7
2017-02-16 20:25:02
set them equal
set them equal
copeland
2017-02-16 20:25:05
We set them equal:\[x^2+a(x-2)(x-3)(x-4)=5x-6.\]Bringing it all to the right-hand side gives\[x^2-5x+6+a(x-2)(x-3)(x-4)=0.\]
We set them equal:\[x^2+a(x-2)(x-3)(x-4)=5x-6.\]Bringing it all to the right-hand side gives\[x^2-5x+6+a(x-2)(x-3)(x-4)=0.\]
copeland
2017-02-16 20:25:06
See anything?
See anything?
duck_master
2017-02-16 20:25:35
factoring!
factoring!
abishek99
2017-02-16 20:25:35
factor first three terms
factor first three terms
Antidifferentiation108
2017-02-16 20:25:35
can factor it
can factor it
ninjataco
2017-02-16 20:25:35
factor x^2-5x+6 as (x-2)(x-3)
factor x^2-5x+6 as (x-2)(x-3)
geogirl08
2017-02-16 20:25:39
(x-2)*(x-3)
(x-2)*(x-3)
PiAreSquared
2017-02-16 20:25:39
Quadratic
Quadratic
brainiac1
2017-02-16 20:25:39
the first few terms equal $(x-2)(x-3)$
the first few terms equal $(x-2)(x-3)$
copeland
2017-02-16 20:25:41
The "quadratic part" factors!
The "quadratic part" factors!
copeland
2017-02-16 20:25:42
\[(x-2)(x-3)+a(x-2)(x-3)(x-4)=0.\]
\[(x-2)(x-3)+a(x-2)(x-3)(x-4)=0.\]
copeland
2017-02-16 20:25:44
By the way, why should we have guessed that the quadratic part would factor like this?
By the way, why should we have guessed that the quadratic part would factor like this?
brainiac1
2017-02-16 20:26:34
2 and 3 are roots
2 and 3 are roots
ninjataco
2017-02-16 20:26:34
the cubic has roots 2 and 3
the cubic has roots 2 and 3
copeland
2017-02-16 20:26:36
The cubic part has roots 2 and 3. The whole thing also has roots 2 and 3. Therefore the quadratic part has roots 2 and 3. We also know its leading term is $x^2$, so that's the only thing it could have been.
The cubic part has roots 2 and 3. The whole thing also has roots 2 and 3. Therefore the quadratic part has roots 2 and 3. We also know its leading term is $x^2$, so that's the only thing it could have been.
copeland
2017-02-16 20:26:38
So what is the third root of this polynomial?
So what is the third root of this polynomial?
copeland
2017-02-16 20:27:04
Actually, just give me an equation that this root solves.
Actually, just give me an equation that this root solves.
copeland
2017-02-16 20:27:08
That'll be just as helpful.
That'll be just as helpful.
duck_master
2017-02-16 20:27:30
4-1/a
4-1/a
yrnsmurf
2017-02-16 20:27:30
4-1/a
4-1/a
Derive_Foiler
2017-02-16 20:27:30
1+a(x-4)?
1+a(x-4)?
ninjataco
2017-02-16 20:27:30
1+a(x-4)=0
1+a(x-4)=0
duck_master
2017-02-16 20:27:30
1+a(x-4)=0
1+a(x-4)=0
brainiac1
2017-02-16 20:27:30
4-1/a
4-1/a
tycooper
2017-02-16 20:27:30
a*(x-4)=-1
a*(x-4)=-1
copeland
2017-02-16 20:27:32
Canceling $(x-2)(x-3)$ gives
Canceling $(x-2)(x-3)$ gives
copeland
2017-02-16 20:27:32
\[1+a(x_{AB}-4)=0.\]
\[1+a(x_{AB}-4)=0.\]
copeland
2017-02-16 20:27:35
That tells us the $x$-value of the third root in terms of $a$. (Remember, our goal here is to use what we know about the roots to find $a$.)
That tells us the $x$-value of the third root in terms of $a$. (Remember, our goal here is to use what we know about the roots to find $a$.)
copeland
2017-02-16 20:27:42
What similar equation will we get for the $x$-value of the third intersection of the line through $AC$?
What similar equation will we get for the $x$-value of the third intersection of the line through $AC$?
copeland
2017-02-16 20:28:13
I'll give you a moment.
I'll give you a moment.
geogirl08
2017-02-16 20:29:17
1+a(xAC - 3) = 0
1+a(xAC - 3) = 0
fractal161
2017-02-16 20:29:17
$1+a(x_{AC}-3)=0$
$1+a(x_{AC}-3)=0$
duck_master
2017-02-16 20:29:17
1+a(x-3)=0
1+a(x-3)=0
kunsun
2017-02-16 20:29:17
1+a(x_AC-3) = 0
1+a(x_AC-3) = 0
GeronimoStilton
2017-02-16 20:29:17
$1 + a(x_{AC} - 3) = 0$
$1 + a(x_{AC} - 3) = 0$
yrnsmurf
2017-02-16 20:29:17
3-1/a
3-1/a
Tribefan
2017-02-16 20:29:17
1+a(x-3)=0
1+a(x-3)=0
copeland
2017-02-16 20:29:19
Going the long way about it, we know that line has slope $\dfrac{16-4}{4-2}=6$, so must be $y=6x-8$.
Going the long way about it, we know that line has slope $\dfrac{16-4}{4-2}=6$, so must be $y=6x-8$.
copeland
2017-02-16 20:29:21
Substituting gives \[x^2+a(x-2)(x-3)(x-4)=6x-8\] so \[(x-2)(x-4)+a(x-2)(x-3)(x-4)=0,\] and
Substituting gives \[x^2+a(x-2)(x-3)(x-4)=6x-8\] so \[(x-2)(x-4)+a(x-2)(x-3)(x-4)=0,\] and
copeland
2017-02-16 20:29:23
\[1+a(x_{AC}-3)=0.\]
\[1+a(x_{AC}-3)=0.\]
copeland
2017-02-16 20:29:26
Doing the same for $BC$ gives
Doing the same for $BC$ gives
copeland
2017-02-16 20:29:30
\[1+a(x_{BC}-2)=0.\]
\[1+a(x_{BC}-2)=0.\]
copeland
2017-02-16 20:29:37
Now what?
Now what?
GeronimoStilton
2017-02-16 20:30:48
Sum them!!!
Sum them!!!
Derive_Foiler
2017-02-16 20:30:48
use the fact that those x's add to 24? (Add the equations together)
use the fact that those x's add to 24? (Add the equations together)
fractal161
2017-02-16 20:30:48
Smash them all together?
Smash them all together?
geogirl08
2017-02-16 20:30:48
xAB + xAC + xBC = 24
xAB + xAC + xBC = 24
yrnsmurf
2017-02-16 20:30:48
9-3/a=24
9-3/a=24
abishek99
2017-02-16 20:30:48
add them up
add them up
Reef334
2017-02-16 20:30:48
We know the sum of the x-coordinates, so solve for a
We know the sum of the x-coordinates, so solve for a
copeland
2017-02-16 20:30:55
When we add these we get \[3+a(x_{AB}+x_{AC}+x_{BC}-9)=0.\]
When we add these we get \[3+a(x_{AB}+x_{AC}+x_{BC}-9)=0.\]
copeland
2017-02-16 20:31:04
We also know that $x_{AB}+x_{AC}+x_{BC}=24$.
We also know that $x_{AB}+x_{AC}+x_{BC}=24$.
copeland
2017-02-16 20:31:06
What is $a$?
What is $a$?
letsgomath
2017-02-16 20:31:33
-1/5
-1/5
DaAsianPotatoe
2017-02-16 20:31:33
-1/5
-1/5
SomethingNeutral
2017-02-16 20:31:33
-1/5
-1/5
ilovemath04
2017-02-16 20:31:33
-1/5
-1/5
MathTechFire
2017-02-16 20:31:33
-1/5
-1/5
copeland
2017-02-16 20:31:35
\[3+15a=0,\] so $a=-\dfrac15$.
\[3+15a=0,\] so $a=-\dfrac15$.
copeland
2017-02-16 20:31:35
And what is $f(0)$?
And what is $f(0)$?
ninjataco
2017-02-16 20:32:27
24/5
24/5
acegikmoqsuwy2000
2017-02-16 20:32:27
$\dfrac {24}5$
$\dfrac {24}5$
summitwei
2017-02-16 20:32:27
24/5
24/5
gradysocool
2017-02-16 20:32:27
24/5
24/5
rt03
2017-02-16 20:32:27
24/5
24/5
ilovemath04
2017-02-16 20:32:27
24/5
24/5
abishek99
2017-02-16 20:32:27
-1/5(-2)(-3)(-4) = 24/5 --> D
-1/5(-2)(-3)(-4) = 24/5 --> D
tdeng
2017-02-16 20:32:27
24/5
24/5
GeronimoStilton
2017-02-16 20:32:27
$D, \frac{24}{5}$
$D, \frac{24}{5}$
yrnsmurf
2017-02-16 20:32:27
24/5=D
24/5=D
Reef334
2017-02-16 20:32:27
-24a = (D) 24/5
-24a = (D) 24/5
Derive_Foiler
2017-02-16 20:32:27
24/5, D
24/5, D
geogirl08
2017-02-16 20:32:27
24/5
24/5
duck_master
2017-02-16 20:32:27
D) 24/5
D) 24/5
anonymous0
2017-02-16 20:32:27
-24a
-24a
copeland
2017-02-16 20:32:29
$f(0)=0^2-\dfrac15(0-2)(0-3)(0-4)=\boxed{\dfrac{24}5}$. (D).
$f(0)=0^2-\dfrac15(0-2)(0-3)(0-4)=\boxed{\dfrac{24}5}$. (D).
copeland
2017-02-16 20:32:42
Ooh, we're getting close. I'm getting all tingly.
Ooh, we're getting close. I'm getting all tingly.
copeland
2017-02-16 20:32:48
Shall we rock on?
Shall we rock on?
SomethingNeutral
2017-02-16 20:33:12
yes!!!!!!
yes!!!!!!
HighQXMoney
2017-02-16 20:33:12
I'm hungry, but sure.
I'm hungry, but sure.
copeland
2017-02-16 20:33:15
24. Quadrilateral $ABCD$ has right angles at $B$ and $C$, $\triangle ABC \sim \triangle BCD$, and $AB > BC$. There is a point $E$ in the interior of $ABCD$ such that $\triangle ABC \sim \triangle CEB$ and the area of $\triangle AED$ is $17$ times the area of $\triangle CEB$. What is $\dfrac{AB}{BC}$?
$\textbf{(A)}\quad 1 + \sqrt{2} \qquad \qquad \textbf{(B)}\quad 2 + \sqrt{2} \qquad\qquad \textbf{(C)}\quad \sqrt{17} \qquad\qquad \textbf{(D)}\quad 2 + \sqrt{5} \qquad\qquad\textbf{(E)}\quad 1 + 2\sqrt{3}$
24. Quadrilateral $ABCD$ has right angles at $B$ and $C$, $\triangle ABC \sim \triangle BCD$, and $AB > BC$. There is a point $E$ in the interior of $ABCD$ such that $\triangle ABC \sim \triangle CEB$ and the area of $\triangle AED$ is $17$ times the area of $\triangle CEB$. What is $\dfrac{AB}{BC}$?
$\textbf{(A)}\quad 1 + \sqrt{2} \qquad \qquad \textbf{(B)}\quad 2 + \sqrt{2} \qquad\qquad \textbf{(C)}\quad \sqrt{17} \qquad\qquad \textbf{(D)}\quad 2 + \sqrt{5} \qquad\qquad\textbf{(E)}\quad 1 + 2\sqrt{3}$
summitwei
2017-02-16 20:33:35
diagram pl0x
diagram pl0x
GeronimoStilton
2017-02-16 20:33:35
Diagram!!!
Diagram!!!
CaptainGeo
2017-02-16 20:33:35
draw a diagram
draw a diagram
yrnsmurf
2017-02-16 20:33:38
draw diagram
draw diagram
copeland
2017-02-16 20:33:44
OK, we start with a right trapezoid where the side lengths decrease by something that looks like a common ratio.
OK, we start with a right trapezoid where the side lengths decrease by something that looks like a common ratio.
copeland
2017-02-16 20:33:45
copeland
2017-02-16 20:33:54
Let's drop some lengths in there. What should we label as our lengths?
Let's drop some lengths in there. What should we label as our lengths?
gradysocool
2017-02-16 20:34:39
BC=1?
BC=1?
First
2017-02-16 20:34:39
WLOG assume $BC=1$
WLOG assume $BC=1$
LaTeX_turtle
2017-02-16 20:34:39
BC=1?
BC=1?
copeland
2017-02-16 20:34:41
That's not bad. Then what should we call $AB$?
That's not bad. Then what should we call $AB$?
ninjataco
2017-02-16 20:35:06
x
x
LaTeX_turtle
2017-02-16 20:35:06
eggs
eggs
LaTeX_turtle
2017-02-16 20:35:06
AB=x
AB=x
CaptainGeo
2017-02-16 20:35:06
x
x
Derive_Foiler
2017-02-16 20:35:06
AB=x. That's what we want to solve for
AB=x. That's what we want to solve for
anonymous0
2017-02-16 20:35:06
x
x
gradysocool
2017-02-16 20:35:06
x of course
x of course
SomethingNeutral
2017-02-16 20:35:06
x
x
copeland
2017-02-16 20:35:07
OK, then if $AB=x$, what is $DC$?
OK, then if $AB=x$, what is $DC$?
Derive_Foiler
2017-02-16 20:35:27
DC=1/x
DC=1/x
abishek99
2017-02-16 20:35:27
1/x
1/x
yrnsmurf
2017-02-16 20:35:27
1/x
1/x
gradysocool
2017-02-16 20:35:27
$\frac1x
$\frac1x
GeronimoStilton
2017-02-16 20:35:27
$\frac{1}{x}$ by similar triangles
$\frac{1}{x}$ by similar triangles
Tribefan
2017-02-16 20:35:27
1/x
1/x
islander7
2017-02-16 20:35:27
1/x
1/x
copeland
2017-02-16 20:35:28
Oh.
Oh.
copeland
2017-02-16 20:35:30
That's terrible.
That's terrible.
copeland
2017-02-16 20:35:33
Let's start over.
Let's start over.
copeland
2017-02-16 20:35:37
New paper.
New paper.
copeland
2017-02-16 20:35:41
copeland
2017-02-16 20:35:59
Give me better lengths, you fraction-wielding psychopaths.
Give me better lengths, you fraction-wielding psychopaths.
abishek99
2017-02-16 20:36:24
CD = 1
CD = 1
Derive_Foiler
2017-02-16 20:36:24
Make BC = x, AB = x^2, and DC = 1?
Make BC = x, AB = x^2, and DC = 1?
SomethingNeutral
2017-02-16 20:36:24
dc = 1, bc = x, ab = x^2
dc = 1, bc = x, ab = x^2
geogirl08
2017-02-16 20:36:24
CD = 1
CD = 1
LaTeX_turtle
2017-02-16 20:36:24
DC=1, to get rid of remainders.
DC=1, to get rid of remainders.
akaashp11
2017-02-16 20:36:24
$AB = x^2$
$AB = x^2$
LaTeX_turtle
2017-02-16 20:36:24
BC=x
BC=x
LaTeX_turtle
2017-02-16 20:36:24
AB=x^2
AB=x^2
gradysocool
2017-02-16 20:36:31
why is that terrible? Fractions are nicer than remembering to divide AB by BC at the end. This way it's just "Find x"
why is that terrible? Fractions are nicer than remembering to divide AB by BC at the end. This way it's just "Find x"
copeland
2017-02-16 20:36:41
If $AB=x^2$ and $BC=x$ then what are we finding?
If $AB=x^2$ and $BC=x$ then what are we finding?
SomethingNeutral
2017-02-16 20:36:54
x
x
geogirl08
2017-02-16 20:36:54
x
x
CaptainGeo
2017-02-16 20:36:54
x
x
duck_master
2017-02-16 20:36:54
x lol
x lol
anonymous0
2017-02-16 20:36:54
x
x
HighQXMoney
2017-02-16 20:36:54
$x$
$x$
copeland
2017-02-16 20:36:55
We're still finding $\dfrac{AB}{BC}=x$.
We're still finding $\dfrac{AB}{BC}=x$.
copeland
2017-02-16 20:37:06
By $\triangle ABC\sim\triangle BCD$, we know that $\dfrac{AB}{BC}=\dfrac{BC}{CD}$. In order to avoid (at least for now) having denominators, let's let $CD=1$ and $BC=x$.
By $\triangle ABC\sim\triangle BCD$, we know that $\dfrac{AB}{BC}=\dfrac{BC}{CD}$. In order to avoid (at least for now) having denominators, let's let $CD=1$ and $BC=x$.
copeland
2017-02-16 20:37:15
Let's drop $E$ in there as well. What do we know that will help in placing $E$?
Let's drop $E$ in there as well. What do we know that will help in placing $E$?
tdeng
2017-02-16 20:38:00
ABC is similar to CEB
ABC is similar to CEB
Derive_Foiler
2017-02-16 20:38:00
Right angles!
Right angles!
duck_master
2017-02-16 20:38:00
CEB and ABC are similar triangles
CEB and ABC are similar triangles
summitwei
2017-02-16 20:38:00
on the semicircle with diameter BC
on the semicircle with diameter BC
LaTeX_turtle
2017-02-16 20:38:00
ABC~CEB
ABC~CEB
GeronimoStilton
2017-02-16 20:38:00
$\triangle ABC \sim \triangle CEB$
$\triangle ABC \sim \triangle CEB$
tdeng
2017-02-16 20:38:00
It is on the semicircle with BC as its diameter
It is on the semicircle with BC as its diameter
LaTeX_turtle
2017-02-16 20:38:00
CEB has right angle at E.
CEB has right angle at E.
copeland
2017-02-16 20:38:05
Great. Since $\triangle ABC\sim\triangle CEB$, we know that $\angle CEB$ is a right angle. We also know that $\dfrac{EC}{BE}=x>1$.
Great. Since $\triangle ABC\sim\triangle CEB$, we know that $\angle CEB$ is a right angle. We also know that $\dfrac{EC}{BE}=x>1$.
copeland
2017-02-16 20:38:06
We'll put it right about here:
We'll put it right about here:
copeland
2017-02-16 20:38:07
copeland
2017-02-16 20:38:21
What other ratio do we know?
What other ratio do we know?
geogirl08
2017-02-16 20:38:57
AED/CEB = 17
AED/CEB = 17
LaTeX_turtle
2017-02-16 20:38:57
[AED] = 17[CEB]
[AED] = 17[CEB]
duck_master
2017-02-16 20:38:57
the area of AED compared to CEB
the area of AED compared to CEB
tdeng
2017-02-16 20:38:57
$\frac{[\triangle AED]}{[\triangle CEB]}$
$\frac{[\triangle AED]}{[\triangle CEB]}$
copeland
2017-02-16 20:39:05
Too soon. We only have a few lengths.
Too soon. We only have a few lengths.
copeland
2017-02-16 20:39:19
That does tell us that our diagram is awful, but let's forget about that for now.
That does tell us that our diagram is awful, but let's forget about that for now.
ninjataco
2017-02-16 20:39:56
CE/BE = x
CE/BE = x
kpatel2000
2017-02-16 20:39:56
AB/BC = CE/BE
AB/BC = CE/BE
cyborg108
2017-02-16 20:39:56
EC/EB = x
EC/EB = x
brainiac1
2017-02-16 20:39:56
we can figure out the side lengths of CEB
we can figure out the side lengths of CEB
yrnsmurf
2017-02-16 20:39:56
find BE and CE
find BE and CE
copeland
2017-02-16 20:40:00
Since $\triangle ABC\sim\triangle CEB$, we know $\dfrac{CE}{EB}=\dfrac{AB}{BC}=x$.
Since $\triangle ABC\sim\triangle CEB$, we know $\dfrac{CE}{EB}=\dfrac{AB}{BC}=x$.
copeland
2017-02-16 20:40:18
We actually know exactly what $EB$ and $EC$ are now. We'll get there in a moment.
We actually know exactly what $EB$ and $EC$ are now. We'll get there in a moment.
copeland
2017-02-16 20:40:22
I want to set up another variable here (it depends on $x$, but the diagram will be clearer with a second variable). Let's let $EB=t$.
I want to set up another variable here (it depends on $x$, but the diagram will be clearer with a second variable). Let's let $EB=t$.
copeland
2017-02-16 20:40:23
That makes $CE=xt$.
That makes $CE=xt$.
copeland
2017-02-16 20:40:24
copeland
2017-02-16 20:40:31
What else should we add to the diagram now so that we can better study $\triangle BEC$?
What else should we add to the diagram now so that we can better study $\triangle BEC$?
math9990
2017-02-16 20:41:25
altitude
altitude
fractal161
2017-02-16 20:41:25
Altitude from $E$ to $BC$?
Altitude from $E$ to $BC$?
copeland
2017-02-16 20:41:28
If we drop the altitude from $E$ to $BC$ we get more similar triangles.
If we drop the altitude from $E$ to $BC$ we get more similar triangles.
copeland
2017-02-16 20:41:29
copeland
2017-02-16 20:41:30
And what is $EP$?
And what is $EP$?
summitwei
2017-02-16 20:42:37
t^2
t^2
ninjataco
2017-02-16 20:42:37
t^2
t^2
brainiac1
2017-02-16 20:42:37
t^2
t^2
letsgomath
2017-02-16 20:42:37
T^2
T^2
geogirl08
2017-02-16 20:42:37
t^2
t^2
cyborg108
2017-02-16 20:42:37
t^2
t^2
letsgomath
2017-02-16 20:42:37
t^2
t^2
abishek99
2017-02-16 20:42:37
t^2
t^2
rt03
2017-02-16 20:42:37
t^2
t^2
DarkPikachu
2017-02-16 20:42:37
t^2
t^2
copeland
2017-02-16 20:42:40
We can compute the area of $\triangle CEB$ in two ways. first, since there's a right angle at $E$, we get \[[\triangle CEB]=\dfrac12\cdot t\cdot xt=\dfrac{xt^2}2.\] However, $EP$ is the altitude to $BC$, so \[[\triangle CEB]=\dfrac12\cdot x\cdot EP,\] so $EP=t^2$.
We can compute the area of $\triangle CEB$ in two ways. first, since there's a right angle at $E$, we get \[[\triangle CEB]=\dfrac12\cdot t\cdot xt=\dfrac{xt^2}2.\] However, $EP$ is the altitude to $BC$, so \[[\triangle CEB]=\dfrac12\cdot x\cdot EP,\] so $EP=t^2$.
copeland
2017-02-16 20:42:57
copeland
2017-02-16 20:43:04
We have two variables in our diagram but they're dependent, so let's figure out what $t$ is. What equation does $t$ solve?
We have two variables in our diagram but they're dependent, so let's figure out what $t$ is. What equation does $t$ solve?
cyborg108
2017-02-16 20:43:56
Pythag on BEC
Pythag on BEC
GeronimoStilton
2017-02-16 20:43:56
$t^2 + x^2t^2 = x^2$
$t^2 + x^2t^2 = x^2$
LaTeX_turtle
2017-02-16 20:43:56
Pythagorean?
Pythagorean?
yrnsmurf
2017-02-16 20:43:56
$t^2(1+x^2)=x^2$
$t^2(1+x^2)=x^2$
duck_master
2017-02-16 20:43:56
$t^2(1+x^2)=x^2$ by pythag and stuff
$t^2(1+x^2)=x^2$ by pythag and stuff
CaptainGeo
2017-02-16 20:43:56
t^2+x^2*t^2=x^2
t^2+x^2*t^2=x^2
abishek99
2017-02-16 20:43:56
pythaogrean theorem on BEC
pythaogrean theorem on BEC
Derive_Foiler
2017-02-16 20:43:56
t^2+x^2t^2=x^2 by pythag
t^2+x^2t^2=x^2 by pythag
LaTeX_turtle
2017-02-16 20:43:56
t^2+(xt)^2=x^2
t^2+(xt)^2=x^2
kpatel2000
2017-02-16 20:43:56
pythagorean theorem on EBC
pythagorean theorem on EBC
geogirl08
2017-02-16 20:43:56
t^2 + x^2t^2 = x^2
t^2 + x^2t^2 = x^2
copeland
2017-02-16 20:43:59
The Pythagorean Theorem on $\triangle BEC$ gives \[t^2+x^2t^2=x^2,\]so
The Pythagorean Theorem on $\triangle BEC$ gives \[t^2+x^2t^2=x^2,\]so
copeland
2017-02-16 20:44:01
\[t^2=\frac{x^2}{1+x^2}.\]
\[t^2=\frac{x^2}{1+x^2}.\]
copeland
2017-02-16 20:44:04
That's cute.
That's cute.
copeland
2017-02-16 20:44:14
OK, so the area of $\triangle CEB$ is $\dfrac{xt^2}2$. How can we compute the area of $\triangle ABE$?
OK, so the area of $\triangle CEB$ is $\dfrac{xt^2}2$. How can we compute the area of $\triangle ABE$?
Royalreter1
2017-02-16 20:45:17
AB*BP/2
AB*BP/2
CaptainGeo
2017-02-16 20:45:17
figuring out BP
figuring out BP
copeland
2017-02-16 20:45:36
Let's find $BP$ since that is the altitude to $E$.
Let's find $BP$ since that is the altitude to $E$.
GeronimoStilton
2017-02-16 20:45:39
Why do we need to know the area of $\triangle ABE$?
Why do we need to know the area of $\triangle ABE$?
copeland
2017-02-16 20:45:44
Oh, that's a great question.
Oh, that's a great question.
copeland
2017-02-16 20:45:48
What should our strategy be here?
What should our strategy be here?
copeland
2017-02-16 20:45:55
We want to find the ratio of some sides.
We want to find the ratio of some sides.
copeland
2017-02-16 20:46:02
We know something about the areas of two of the triangles.
We know something about the areas of two of the triangles.
copeland
2017-02-16 20:46:19
It feels like we need to invoke areas. But why would we want to think about the areas of those other two triangles?
It feels like we need to invoke areas. But why would we want to think about the areas of those other two triangles?
BillZhuo
2017-02-16 20:47:11
just find the area of AED by subtracting pieces out of the trapezoid
just find the area of AED by subtracting pieces out of the trapezoid
GeronimoStilton
2017-02-16 20:47:11
We want to find the area of $\triangle ADE$ relative to the area of $\triangle BEC$.
We want to find the area of $\triangle ADE$ relative to the area of $\triangle BEC$.
cyborg108
2017-02-16 20:47:11
For a final equation for the trapezoid?
For a final equation for the trapezoid?
abishek99
2017-02-16 20:47:11
this helps us find the area of AED
this helps us find the area of AED
eswa2000
2017-02-16 20:47:11
subtract the 3 triangles' areas from the quadrilateral to get [AED]
subtract the 3 triangles' areas from the quadrilateral to get [AED]
copeland
2017-02-16 20:47:13
We can easily compute the area of the full trapezoid and it depends only on $x$, so maybe we'll get two expressions for that area and that will take us home.
We can easily compute the area of the full trapezoid and it depends only on $x$, so maybe we'll get two expressions for that area and that will take us home.
copeland
2017-02-16 20:47:26
Now we're finding $BP$. What similarity gives us $BP$?
Now we're finding $BP$. What similarity gives us $BP$?
gradysocool
2017-02-16 20:48:53
BEP~BEC
BEP~BEC
Derive_Foiler
2017-02-16 20:48:53
use triangle BEP and BCE or CBA
use triangle BEP and BCE or CBA
LaTeX_turtle
2017-02-16 20:48:53
BPE~BEC
BPE~BEC
copeland
2017-02-16 20:49:03
There are a billion similarities now: $\triangle BPE\sim\triangle BEC\sim\triangle CBA$, etc. And what is $BP$?
There are a billion similarities now: $\triangle BPE\sim\triangle BEC\sim\triangle CBA$, etc. And what is $BP$?
GeronimoStilton
2017-02-16 20:49:59
$\frac{t^2}{x}$
$\frac{t^2}{x}$
gradysocool
2017-02-16 20:49:59
$\frac{t^2}{x}$
$\frac{t^2}{x}$
Reef334
2017-02-16 20:49:59
t^2/x
t^2/x
LaTeX_turtle
2017-02-16 20:49:59
Gives $\frac{BP}{t} = \frac{t}{x}$
Gives $\frac{BP}{t} = \frac{t}{x}$
copeland
2017-02-16 20:50:01
Since $\dfrac{BP}{PE}=\dfrac1x$ and $PE=t^2$, we get $BP=\dfrac{t^2}x$.
Since $\dfrac{BP}{PE}=\dfrac1x$ and $PE=t^2$, we get $BP=\dfrac{t^2}x$.
copeland
2017-02-16 20:50:02
And what is $PC$?
And what is $PC$?
BXU65
2017-02-16 20:52:02
(x^2-t^2)/x
(x^2-t^2)/x
yrnsmurf
2017-02-16 20:52:02
xt^2
xt^2
Skittlesftw
2017-02-16 20:52:02
$ xt^2 $
$ xt^2 $
GeronimoStilton
2017-02-16 20:52:02
$x - \frac{t^2}{x}$
$x - \frac{t^2}{x}$
letsgomath
2017-02-16 20:52:02
(x^2-t^2)/x
(x^2-t^2)/x
mathfever
2017-02-16 20:52:02
xt^2
xt^2
Smoothfang
2017-02-16 20:52:02
xt^2
xt^2
mathman3880
2017-02-16 20:52:02
xt^2
xt^2
copeland
2017-02-16 20:52:20
We can subtract to get \[PC=BC-BP=x-\frac{t^2}x=\frac{x^2-t^2}{x}=\frac{x^2t^2}{x}=xt^2.\]
We can subtract to get \[PC=BC-BP=x-\frac{t^2}x=\frac{x^2-t^2}{x}=\frac{x^2t^2}{x}=xt^2.\]
copeland
2017-02-16 20:52:24
We can also use similarity to get $\dfrac{PC}{t^2}=x$, so $PC=xt^2$.
We can also use similarity to get $\dfrac{PC}{t^2}=x$, so $PC=xt^2$.
copeland
2017-02-16 20:52:33
copeland
2017-02-16 20:52:37
OK, so what is the area of $\triangle ABE$ now?
OK, so what is the area of $\triangle ABE$ now?
Derive_Foiler
2017-02-16 20:53:18
t^2x/2
t^2x/2
ninjataco
2017-02-16 20:53:18
xt^2/2
xt^2/2
Smoothfang
2017-02-16 20:53:18
xt^2/2
xt^2/2
geogirl08
2017-02-16 20:53:18
xt^2/2
xt^2/2
Skittlesftw
2017-02-16 20:53:18
$ \frac{xt^2}{2} $
$ \frac{xt^2}{2} $
BXU65
2017-02-16 20:53:18
xt^2/2
xt^2/2
copeland
2017-02-16 20:53:21
The base is $AB=x^2$ and the height is $\dfrac{t^2}x$, so the area is $\dfrac{xt^2}2$.
The base is $AB=x^2$ and the height is $\dfrac{t^2}x$, so the area is $\dfrac{xt^2}2$.
copeland
2017-02-16 20:53:22
What other area do we get?
What other area do we get?
mathman3880
2017-02-16 20:54:41
[BEC]=[ECD] = xt^2/2
[BEC]=[ECD] = xt^2/2
duck_master
2017-02-16 20:54:41
CEB
CEB
tarzanjunior
2017-02-16 20:54:41
ECD
ECD
MP2016p42
2017-02-16 20:54:41
EDC
EDC
Derive_Foiler
2017-02-16 20:54:41
CED, which is xt^2/2 hmmmm . . .
CED, which is xt^2/2 hmmmm . . .
BXU65
2017-02-16 20:54:41
Triangle DEC
Triangle DEC
summitwei
2017-02-16 20:54:41
[CDE]=xt^2/2
[CDE]=xt^2/2
MP2016p42
2017-02-16 20:54:41
EDC area = x^2t/2
EDC area = x^2t/2
cyborg108
2017-02-16 20:54:41
EBC = x*t^2 / 2
EBC = x*t^2 / 2
copeland
2017-02-16 20:54:44
Triangle $DCE$ has base $CD=1$ and height $xt^2$, so has area $\dfrac{xt^2}2$.
Triangle $DCE$ has base $CD=1$ and height $xt^2$, so has area $\dfrac{xt^2}2$.
copeland
2017-02-16 20:54:48
Huh, that looks familiar. Do we know the area of $\triangle ADE$?
Huh, that looks familiar. Do we know the area of $\triangle ADE$?
BXU65
2017-02-16 20:56:38
17xt^2/2
17xt^2/2
letsgomath
2017-02-16 20:56:38
17 times the area of CEB
17 times the area of CEB
Skittlesftw
2017-02-16 20:56:38
$ 17 \cdot triangle CEB $
$ 17 \cdot triangle CEB $
copeland
2017-02-16 20:56:40
It's given to us that it has area $\dfrac{17xt^2}2$.
It's given to us that it has area $\dfrac{17xt^2}2$.
copeland
2017-02-16 20:56:41
Let's put those areas in there.
Let's put those areas in there.
copeland
2017-02-16 20:56:42
I'm going to write $\dfrac{xt^2}2$ as $a$:
I'm going to write $\dfrac{xt^2}2$ as $a$:
copeland
2017-02-16 20:56:47
copeland
2017-02-16 20:56:52
The total area is $20a=10xt^2$. Do we have another expression for the total area?
The total area is $20a=10xt^2$. Do we have another expression for the total area?
duck_master
2017-02-16 20:57:51
$\frac{(x^2+1)x}{2}$
$\frac{(x^2+1)x}{2}$
ninjataco
2017-02-16 20:57:51
x(1+x^2)/2
x(1+x^2)/2
Reef334
2017-02-16 20:57:51
x(x^2+1)/2
x(x^2+1)/2
SomethingNeutral
2017-02-16 20:57:51
(1+x^2)x/2
(1+x^2)x/2
summitwei
2017-02-16 20:57:51
(1+x^2)*x/2
(1+x^2)*x/2
GeronimoStilton
2017-02-16 20:57:51
$\frac{x^3 + x}{2}$
$\frac{x^3 + x}{2}$
BXU65
2017-02-16 20:57:51
(x^3+x)/2
(x^3+x)/2
geogirl08
2017-02-16 20:57:51
(x^2+1)x/2
(x^2+1)x/2
CrystalEye
2017-02-16 20:57:51
(1+x^2)x/2
(1+x^2)x/2
brainiac1
2017-02-16 20:57:51
(1+x^2)x/2
(1+x^2)x/2
Tribefan
2017-02-16 20:57:51
(x^2+1)/2x
(x^2+1)/2x
copeland
2017-02-16 20:57:53
The region is a right trapezoid so it has area $\dfrac{1+x^2}2\cdot x$.
The region is a right trapezoid so it has area $\dfrac{1+x^2}2\cdot x$.
copeland
2017-02-16 20:57:57
Equating these gives \[10xt^2=\frac{x(1+x^2)}2,\]and substituting $t^2=\dfrac{x^2}{1+x^2}$ and simplifying, we get
Equating these gives \[10xt^2=\frac{x(1+x^2)}2,\]and substituting $t^2=\dfrac{x^2}{1+x^2}$ and simplifying, we get
copeland
2017-02-16 20:58:04
\[20x^2=x^4+2x^2+1.\]
\[20x^2=x^4+2x^2+1.\]
brainiac1
2017-02-16 20:58:20
it's a quadratic!
it's a quadratic!
copeland
2017-02-16 20:58:21
Go us!
Go us!
copeland
2017-02-16 20:59:47
What's $x^2$?
What's $x^2$?
duck_master
2017-02-16 21:00:41
$9+\sqrt{80}$, $9-\sqrt{80}$
$9+\sqrt{80}$, $9-\sqrt{80}$
BXU65
2017-02-16 21:00:41
9+4sqrt(5)
9+4sqrt(5)
celestialphoenix3768
2017-02-16 21:00:41
quadratic formula
quadratic formula
Skittlesftw
2017-02-16 21:00:41
$ 9+4\sqrt5 $????
$ 9+4\sqrt5 $????
BXU65
2017-02-16 21:00:41
9 + 4sqrt(5)
9 + 4sqrt(5)
letsgomath
2017-02-16 21:00:41
9 plus/minus 4sqrt5
9 plus/minus 4sqrt5
Reef334
2017-02-16 21:00:41
9+4sqrt(5)
9+4sqrt(5)
SomethingNeutral
2017-02-16 21:00:41
9$\pm$4sqrt5
9$\pm$4sqrt5
copeland
2017-02-16 21:00:51
Wow, there are two solutions.
Wow, there are two solutions.
copeland
2017-02-16 21:01:03
Interestingly, they are reciprocals of one another.
Interestingly, they are reciprocals of one another.
copeland
2017-02-16 21:01:10
What happens if we choose the little one?
What happens if we choose the little one?
cyborg108
2017-02-16 21:02:11
AB < BC
AB < BC
cyborg108
2017-02-16 21:02:11
contradiction with the givens
contradiction with the givens
summitwei
2017-02-16 21:02:11
then x<1 which is obviously wrong
then x<1 which is obviously wrong
GeronimoStilton
2017-02-16 21:02:11
Then $x < 1$
Then $x < 1$
BXU65
2017-02-16 21:02:11
x would be 2- sqrt(5)
x would be 2- sqrt(5)
howie2000
2017-02-16 21:02:11
x<1 so AB<BC
x<1 so AB<BC
yrnsmurf
2017-02-16 21:02:11
you get the smaller side
you get the smaller side
copeland
2017-02-16 21:02:13
If we choose the little one, then $AB<1$ and the diagram reflects. Cool. The little value is wrong, but it tells us about a symmetry to the problem that we knew about (and that the problem statement works to break).
If we choose the little one, then $AB<1$ and the diagram reflects. Cool. The little value is wrong, but it tells us about a symmetry to the problem that we knew about (and that the problem statement works to break).
copeland
2017-02-16 21:02:22
By the quadratic formula, we get $x^2=\dfrac{18\pm\sqrt{18^2-4}}2=9\pm4\sqrt5$.
By the quadratic formula, we get $x^2=\dfrac{18\pm\sqrt{18^2-4}}2=9\pm4\sqrt5$.
copeland
2017-02-16 21:02:35
We need $x>1$ so $x^2=9\pm4\sqrt5$.
We need $x>1$ so $x^2=9\pm4\sqrt5$.
copeland
2017-02-16 21:02:37
So what must the answer be?
So what must the answer be?
ninjataco
2017-02-16 21:03:17
D
D
BXU65
2017-02-16 21:03:17
2 + sqrt(5) or D
2 + sqrt(5) or D
tdeng
2017-02-16 21:03:17
D
D
GeronimoStilton
2017-02-16 21:03:17
$2+\sqrt{5}$, D
$2+\sqrt{5}$, D
mathman3880
2017-02-16 21:03:17
2+sqrt5
2+sqrt5
celestialphoenix3768
2017-02-16 21:03:17
D
D
nukelauncher
2017-02-16 21:03:17
2+sqrt5 so D
2+sqrt5 so D
MathTechFire
2017-02-16 21:03:17
D?
D?
yrnsmurf
2017-02-16 21:03:17
2+sqrt5=
2+sqrt5=
cyborg108
2017-02-16 21:03:17
2+sqrt(5)
2+sqrt(5)
gradysocool
2017-02-16 21:03:19
well, it's gotta have a $\sqrt{5}$ so...
well, it's gotta have a $\sqrt{5}$ so...
copeland
2017-02-16 21:03:22
The only answer choice that is friends with $\sqrt5$ is (D), $\boxed{2+\sqrt5}$.
The only answer choice that is friends with $\sqrt5$ is (D), $\boxed{2+\sqrt5}$.
copeland
2017-02-16 21:03:31
Indeed, if we square this value we get \[4+5+2\cdot2\sqrt5=9+4\sqrt5,\] so D is indeed the answer.
Indeed, if we square this value we get \[4+5+2\cdot2\sqrt5=9+4\sqrt5,\] so D is indeed the answer.
copeland
2017-02-16 21:03:41
If you're curious, $\dfrac{BC}{BP}=1+x^2\approx19$, so $E$ is really close to $\overline{AB}$ and diagram to scale looks like this:
If you're curious, $\dfrac{BC}{BP}=1+x^2\approx19$, so $E$ is really close to $\overline{AB}$ and diagram to scale looks like this:
copeland
2017-02-16 21:03:48
GeronimoStilton
2017-02-16 21:04:16
That's a harder diagram to work with.
That's a harder diagram to work with.
copeland
2017-02-16 21:04:25
It's hard to believe those triangles have the same area, too.
It's hard to believe those triangles have the same area, too.
copeland
2017-02-16 21:04:39
You know those experiments with the little kids and the water? I feel like that.
You know those experiments with the little kids and the water? I feel like that.
copeland
2017-02-16 21:04:43
(I'm the kid in the analogy.)
(I'm the kid in the analogy.)
GeronimoStilton
2017-02-16 21:04:51
Final question!!!
Final question!!!
gradysocool
2017-02-16 21:04:55
Why is [ABE]=[BCE]=[CDE] ? Is there an easy way to get this?
Why is [ABE]=[BCE]=[CDE] ? Is there an easy way to get this?
copeland
2017-02-16 21:04:59
Oh, yeah. I don't now.
Oh, yeah. I don't now.
copeland
2017-02-16 21:05:04
It's cool, though.
It's cool, though.
copeland
2017-02-16 21:05:14
Also, you're saying what we did wasn't easy?
Also, you're saying what we did wasn't easy?
copeland
2017-02-16 21:05:43
Here we go.
Here we go.
copeland
2017-02-16 21:05:46
25. A set of $n$ people participate in an online video basketball tournament. Each person may be a member of any number of 5-player teams, but no two teams may have exactly the same 5 members. The site statistics show a curious fact: The average, over all subsets of size 9 of the set of $n$ participants, of the number of complete teams whose members are among those 9 people is equal to the reciprocal of the average, over all subsets of size 8 of the set of $n$ participants, of the number of complete teams whose members are among those 8 people. How many values $n$, $9\leq n\leq 2017$, can be the number of participants?
$\textbf{(A)}\quad477 \qquad \qquad \textbf{(B)}\quad 482 \qquad\qquad \textbf{(C)}\quad 487 \qquad\qquad \textbf{(D)}\quad 557 \qquad\qquad\textbf{(E)}\quad 562$
25. A set of $n$ people participate in an online video basketball tournament. Each person may be a member of any number of 5-player teams, but no two teams may have exactly the same 5 members. The site statistics show a curious fact: The average, over all subsets of size 9 of the set of $n$ participants, of the number of complete teams whose members are among those 9 people is equal to the reciprocal of the average, over all subsets of size 8 of the set of $n$ participants, of the number of complete teams whose members are among those 8 people. How many values $n$, $9\leq n\leq 2017$, can be the number of participants?
$\textbf{(A)}\quad477 \qquad \qquad \textbf{(B)}\quad 482 \qquad\qquad \textbf{(C)}\quad 487 \qquad\qquad \textbf{(D)}\quad 557 \qquad\qquad\textbf{(E)}\quad 562$
copeland
2017-02-16 21:06:02
Alright, so that's a bucketful of words. Great. Usually I budget extra time to read the geometry problems.
Alright, so that's a bucketful of words. Great. Usually I budget extra time to read the geometry problems.
copeland
2017-02-16 21:06:31
Let's try to get a grip on this problem. We have a bunch of teams. We want to compute some average of something. We do that twice. Let's figure out what that is.
Let's try to get a grip on this problem. We have a bunch of teams. We want to compute some average of something. We do that twice. Let's figure out what that is.
copeland
2017-02-16 21:06:37
An average is just a sum of something divided by the number of things we're summing over.
An average is just a sum of something divided by the number of things we're summing over.
copeland
2017-02-16 21:06:39
Let's unpack this statement:
"The average, over all subsets of size 9 of the set of $n$ participants, of the number of complete teams whose members are among those 9 people"
Let's unpack this statement:
"The average, over all subsets of size 9 of the set of $n$ participants, of the number of complete teams whose members are among those 9 people"
copeland
2017-02-16 21:06:57
So we're taking an average over all the 9-element subsets. How many of those are there?
So we're taking an average over all the 9-element subsets. How many of those are there?
BXU65
2017-02-16 21:08:06
N choose 9
N choose 9
First
2017-02-16 21:08:06
$n C 9$
$n C 9$
geogirl08
2017-02-16 21:08:06
nC9
nC9
Jfault
2017-02-16 21:08:06
$\binom{n}{9}$?
$\binom{n}{9}$?
nukelauncher
2017-02-16 21:08:06
nchoose9
nchoose9
ninjataco
2017-02-16 21:08:06
nC9
nC9
duck_master
2017-02-16 21:08:06
(n choose 9)
(n choose 9)
summitwei
2017-02-16 21:08:06
nC9
nC9
CrystalEye
2017-02-16 21:08:06
nC9
nC9
copeland
2017-02-16 21:08:09
There are $\dbinom n9$ subsets of size 9. That's the denominator.
There are $\dbinom n9$ subsets of size 9. That's the denominator.
copeland
2017-02-16 21:08:14
Now we want to average the number of complete teams that each of these subsets contains. That means, the numerator is the sum of all these numbers as the size-9 subset varies.
Now we want to average the number of complete teams that each of these subsets contains. That means, the numerator is the sum of all these numbers as the size-9 subset varies.
copeland
2017-02-16 21:08:42
How many subsets is a team contained in?
How many subsets is a team contained in?
copeland
2017-02-16 21:09:54
For example, let $n=100$. You, me, your aunt Tracy, Ned from around the block, and some dude dressed like a banana form a team. How many subsets have the five of us in them?
For example, let $n=100$. You, me, your aunt Tracy, Ned from around the block, and some dude dressed like a banana form a team. How many subsets have the five of us in them?
geogirl08
2017-02-16 21:10:45
n-5C4?
n-5C4?
GeronimoStilton
2017-02-16 21:10:45
$\binom{n-5}{4}$
$\binom{n-5}{4}$
az1326
2017-02-16 21:10:45
((n-5)choose 4)
((n-5)choose 4)
brainiac1
2017-02-16 21:10:45
n-5 choose 4
n-5 choose 4
geogirl08
2017-02-16 21:10:45
95C4
95C4
GeronimoStilton
2017-02-16 21:10:45
$\binom{95}{4}$
$\binom{95}{4}$
Reef334
2017-02-16 21:10:45
95 C 4
95 C 4
abishek99
2017-02-16 21:10:45
95C4
95C4
az1326
2017-02-16 21:10:45
(95 choose 4)
(95 choose 4)
gradysocool
2017-02-16 21:10:45
95C4
95C4
First
2017-02-16 21:10:45
95C5?
95C5?
Jfault
2017-02-16 21:10:45
$\binom{95}{4}$
$\binom{95}{4}$
CrystalEye
2017-02-16 21:10:45
(n-5)C4
(n-5)C4
copeland
2017-02-16 21:10:47
We just have to pick 4 more people to make a subset. There are 95 people to choose from.
We just have to pick 4 more people to make a subset. There are 95 people to choose from.
copeland
2017-02-16 21:11:02
Each team contributes 1 to the value for each subset it is contained in. A team of 5 can be filled out to a subset of size 9 by choosing $\dbinom{n-5}4$ other members. Therefore each team contributes exactly $\dbinom{n-5}4$ to the numerator.
Each team contributes 1 to the value for each subset it is contained in. A team of 5 can be filled out to a subset of size 9 by choosing $\dbinom{n-5}4$ other members. Therefore each team contributes exactly $\dbinom{n-5}4$ to the numerator.
copeland
2017-02-16 21:11:10
If there are $t$ teams, then what is the average?
If there are $t$ teams, then what is the average?
geogirl08
2017-02-16 21:13:10
t(n-5C4)/(nC9)
t(n-5C4)/(nC9)
BXU65
2017-02-16 21:13:10
t * n-5 choose 4 / n choose 5
t * n-5 choose 4 / n choose 5
GeronimoStilton
2017-02-16 21:13:10
$\frac{t\binom{n-5}{4}}{\binom{n}{9}}$
$\frac{t\binom{n-5}{4}}{\binom{n}{9}}$
BXU65
2017-02-16 21:13:10
t * n-5 choose 4 / n choose 9
t * n-5 choose 4 / n choose 9
az1326
2017-02-16 21:13:10
t ((n-5) choose 4) / (n choose 9)
t ((n-5) choose 4) / (n choose 9)
duck_master
2017-02-16 21:13:10
$t \frac{\binom{n-5}{4}}{\binom{n}{9}}$
$t \frac{\binom{n-5}{4}}{\binom{n}{9}}$
ninjataco
2017-02-16 21:13:10
t*(n-5)C4 / nC9
t*(n-5)C4 / nC9
copeland
2017-02-16 21:13:13
The average blah blah size 9 blah blah blah is then\[\frac{t\dbinom{n-5}{4}}{\dbinom n9}.\]
The average blah blah size 9 blah blah blah is then\[\frac{t\dbinom{n-5}{4}}{\dbinom n9}.\]
ilikepie2003
2017-02-16 21:13:17
how do you know that the dude isn't a real banana
how do you know that the dude isn't a real banana
copeland
2017-02-16 21:13:21
He may be. Either way, he's an online video basketball phenom.
He may be. Either way, he's an online video basketball phenom.
copeland
2017-02-16 21:13:43
Oh, by the way, you have 15 seconds to count the number of times "of" appears in that one long sentence. Go!
Oh, by the way, you have 15 seconds to count the number of times "of" appears in that one long sentence. Go!
Jfault
2017-02-16 21:14:41
7?
7?
geogirl08
2017-02-16 21:14:41
8
8
letsgomath
2017-02-16 21:14:41
7
7
leonlzg
2017-02-16 21:14:41
11?
11?
Derive_Foiler
2017-02-16 21:14:41
11?
11?
GeronimoStilton
2017-02-16 21:14:41
$13$
$13$
Jfault
2017-02-16 21:14:41
\boxed{9}!
\boxed{9}!
copeland
2017-02-16 21:14:47
Yeah. Who knows?!!?
Yeah. Who knows?!!?
Reef334
2017-02-16 21:14:54
15?
15?
duck_master
2017-02-16 21:14:54
12
12
copeland
2017-02-16 21:14:57
We think it's 11.
We think it's 11.
copeland
2017-02-16 21:15:04
But counting them is nondeterministic.
But counting them is nondeterministic.
copeland
2017-02-16 21:15:09
What is the reciprocal of the average, over all subsets of size 8 of the set of $n$ participants, of the number of complete teams whose members are among those 8 people?
What is the reciprocal of the average, over all subsets of size 8 of the set of $n$ participants, of the number of complete teams whose members are among those 8 people?
shakeNbake
2017-02-16 21:16:49
16
16
ProGameXD
2017-02-16 21:16:49
21
21
amburger66
2017-02-16 21:16:49
eleven!!11!11
eleven!!11!11
BXU65
2017-02-16 21:16:49
7?
7?
mathwiz0803
2017-02-16 21:16:49
18
18
shakeNbake
2017-02-16 21:16:49
oh its 15 just ctrl F
oh its 15 just ctrl F
Haphazard
2017-02-16 21:16:49
there are 16 'of'
there are 16 'of'
BXU65
2017-02-16 21:16:53
t * (n-5 choose 3) / n choose 8
t * (n-5 choose 3) / n choose 8
gradysocool
2017-02-16 21:16:53
$\frac{\binom{n}{8}}{t\binom{n-5}{3}}$
$\frac{\binom{n}{8}}{t\binom{n-5}{3}}$
duck_master
2017-02-16 21:16:53
$\frac{\binom{n}{8}}{t \binom{n-5}{3}}$
$\frac{\binom{n}{8}}{t \binom{n-5}{3}}$
GeronimoStilton
2017-02-16 21:16:53
$$\frac{\binom{n}{8}}{t\binom{n-5}{3}}.$$
$$\frac{\binom{n}{8}}{t\binom{n-5}{3}}.$$
BXU65
2017-02-16 21:16:53
n choose 8 / t * n - 5 choose 3
n choose 8 / t * n - 5 choose 3
copeland
2017-02-16 21:16:55
To fill out a group of 8 beginning with a team of size 5 we need to pick 3 more members from the remaining $n-5$.
To fill out a group of 8 beginning with a team of size 5 we need to pick 3 more members from the remaining $n-5$.
copeland
2017-02-16 21:16:56
The reciprocal of blah average blah blah 8 blah is \[\frac{\dbinom n8}{t\dbinom{n-5}{3}}.\]
The reciprocal of blah average blah blah 8 blah is \[\frac{\dbinom n8}{t\dbinom{n-5}{3}}.\]
copeland
2017-02-16 21:17:01
Setting them equal gives
Setting them equal gives
copeland
2017-02-16 21:17:06
\[\frac{t\dbinom{n-5}{4}}{\dbinom n9}=\frac{\dbinom n8}{t\dbinom{n-5}{3}}.\]
\[\frac{t\dbinom{n-5}{4}}{\dbinom n9}=\frac{\dbinom n8}{t\dbinom{n-5}{3}}.\]
copeland
2017-02-16 21:17:17
Now what?
Now what?
duck_master
2017-02-16 21:17:57
cross multiply
cross multiply
BXU65
2017-02-16 21:17:57
cross multiply?
cross multiply?
Derive_Foiler
2017-02-16 21:17:57
expand . . . ?
expand . . . ?
celestialphoenix3768
2017-02-16 21:17:57
bash bash bash
bash bash bash
SaltySnail2
2017-02-16 21:17:57
cross multiply?
cross multiply?
EulerMacaroni
2017-02-16 21:17:57
cancel like crazy
cancel like crazy
ninjataco
2017-02-16 21:17:57
cross multiply
cross multiply
brainiac1
2017-02-16 21:17:57
now we expand and cancel all the factorials and pray it looks pretty
now we expand and cancel all the factorials and pray it looks pretty
copeland
2017-02-16 21:18:05
Maybe I've got something wrong with me, but I kinda feel like expanding those binomial coefficients.
Maybe I've got something wrong with me, but I kinda feel like expanding those binomial coefficients.
copeland
2017-02-16 21:18:15
Actually, this does make sense. When we see a quotient of two similar binomial coefficients, we can get a lot of cancellation by expanding both of them. So let's expand all our binomial coefficients. What happens to the left when we expand?
Actually, this does make sense. When we see a quotient of two similar binomial coefficients, we can get a lot of cancellation by expanding both of them. So let's expand all our binomial coefficients. What happens to the left when we expand?
copeland
2017-02-16 21:18:34
(We'll cross-multiply in a moment. I love cross-multiplication.)
(We'll cross-multiply in a moment. I love cross-multiplication.)
BXU65
2017-02-16 21:20:54
we get 9 * 8 * 7 * 6 * 5 * t / n * n-1 * n-2 * n-3 * n-4
we get 9 * 8 * 7 * 6 * 5 * t / n * n-1 * n-2 * n-3 * n-4
duck_master
2017-02-16 21:20:54
t$\frac{9!(n-5)!}{4!n!}$
t$\frac{9!(n-5)!}{4!n!}$
brainiac1
2017-02-16 21:20:54
$\frac{15120}{n(n-1)(n-2)(n-3)(n-4)}$
$\frac{15120}{n(n-1)(n-2)(n-3)(n-4)}$
copeland
2017-02-16 21:20:59
On the left we get \[\frac{t\dbinom{n-5}{4}}{\dbinom n9}=\frac{t(n-5)!9!(n-9)!}{n!4!(n-9)!}=\frac{t(n-5)!9!}{n!4!}.\]
On the left we get \[\frac{t\dbinom{n-5}{4}}{\dbinom n9}=\frac{t(n-5)!9!(n-9)!}{n!4!(n-9)!}=\frac{t(n-5)!9!}{n!4!}.\]
copeland
2017-02-16 21:21:02
On the right we get \[\frac{n!3!}{t(n-5)!8!}.\]
On the right we get \[\frac{n!3!}{t(n-5)!8!}.\]
copeland
2017-02-16 21:21:03
Now what are we looking for?
Now what are we looking for?
GeronimoStilton
2017-02-16 21:21:38
The possible values of $n$
The possible values of $n$
Derive_Foiler
2017-02-16 21:21:38
all possible n?
all possible n?
xayy
2017-02-16 21:21:38
n
n
copeland
2017-02-16 21:21:39
Which $n$? What makes $n$ special?
Which $n$? What makes $n$ special?
copeland
2017-02-16 21:23:53
We have\[\frac{t(n-5)!9!}{n!4!}=\frac{n!3!}{t(n-5)!8!}.\]Isolating $t^2$ gives\[t^2=\frac{n!^23!4!}{(n-5)!^28!9!}.\]
We have\[\frac{t(n-5)!9!}{n!4!}=\frac{n!3!}{t(n-5)!8!}.\]Isolating $t^2$ gives\[t^2=\frac{n!^23!4!}{(n-5)!^28!9!}.\]
copeland
2017-02-16 21:23:56
So what makes $n$ special?
So what makes $n$ special?
linqaszayi
2017-02-16 21:24:47
t is an integer so n must satisfy some division conditions
t is an integer so n must satisfy some division conditions
GeronimoStilton
2017-02-16 21:24:47
There exists an integer value of $t$
There exists an integer value of $t$
yrnsmurf
2017-02-16 21:24:47
the ones that make t an integer
the ones that make t an integer
copeland
2017-02-16 21:24:54
We're looking for all the values of $n$ such that $t$ is an integer.
We're looking for all the values of $n$ such that $t$ is an integer.
copeland
2017-02-16 21:25:04
Now let's just go crazy and totally expand that. It really isn't that many terms after we cancel:
Now let's just go crazy and totally expand that. It really isn't that many terms after we cancel:
copeland
2017-02-16 21:25:05
\begin{align*}
t^2
&=\frac{\left[n(n-1)(n-2)(n-3)(n-4)\right]^2}{9\cdot8^2\cdot7^2\cdot6^2\cdot5^2\cdot4}\\
&=\left(\frac{n(n-1)(n-2)(n-3)(n-4)}{8\cdot7\cdot6\cdot5\cdot3\cdot2}\right)^2.\end{align*}
\begin{align*}
t^2
&=\frac{\left[n(n-1)(n-2)(n-3)(n-4)\right]^2}{9\cdot8^2\cdot7^2\cdot6^2\cdot5^2\cdot4}\\
&=\left(\frac{n(n-1)(n-2)(n-3)(n-4)}{8\cdot7\cdot6\cdot5\cdot3\cdot2}\right)^2.\end{align*}
copeland
2017-02-16 21:25:12
Oh my. That's a perfect square. So we're looking for every value of $n$ that makes \[t=\frac{n(n-1)(n-2)(n-3)(n-4)}{8\cdot7\cdot6\cdot5\cdot3\cdot2}\]an integer.
Oh my. That's a perfect square. So we're looking for every value of $n$ that makes \[t=\frac{n(n-1)(n-2)(n-3)(n-4)}{8\cdot7\cdot6\cdot5\cdot3\cdot2}\]an integer.
copeland
2017-02-16 21:25:55
Guess what? We don't know that the right side is an integer. That's what we're exploring! Let's start simple. What do we know the numerator is divisible by?
Guess what? We don't know that the right side is an integer. That's what we're exploring! Let's start simple. What do we know the numerator is divisible by?
nukelauncher
2017-02-16 21:26:39
2, 3, 4, and 5
2, 3, 4, and 5
daniellionyang
2017-02-16 21:26:39
5
5
tdeng
2017-02-16 21:26:39
5*3*2
5*3*2
brainiac1
2017-02-16 21:26:39
it's divisible by 3, 4, and 5
it's divisible by 3, 4, and 5
EulerMacaroni
2017-02-16 21:26:39
5
5
copeland
2017-02-16 21:26:41
We know for sure that the numerator is divisible by 5 since it's the product of 5 consecutive integers.
We know for sure that the numerator is divisible by 5 since it's the product of 5 consecutive integers.
copeland
2017-02-16 21:26:42
Is it divisible by 7?
Is it divisible by 7?
ninjataco
2017-02-16 21:26:58
maybe
maybe
summitwei
2017-02-16 21:26:58
not necessarily
not necessarily
BXU65
2017-02-16 21:26:58
not necessarily
not necessarily
copeland
2017-02-16 21:27:00
Not necessarily. For what $n$ is this divisible by 7?
Not necessarily. For what $n$ is this divisible by 7?
EulerMacaroni
2017-02-16 21:27:44
slso $7$ is $n\equiv 0,1,2,3,4 \pmod 7$
slso $7$ is $n\equiv 0,1,2,3,4 \pmod 7$
nukelauncher
2017-02-16 21:27:44
n = 0, 1, 2, 3, 4 mod 7
n = 0, 1, 2, 3, 4 mod 7
brainiac1
2017-02-16 21:27:44
n = 0, 1, 2, 3, 4 mod 7
n = 0, 1, 2, 3, 4 mod 7
xayy
2017-02-16 21:27:44
0,1,2,3,4 mod 7
0,1,2,3,4 mod 7
duck_master
2017-02-16 21:27:44
n is 0,1,2,3,4 mod 7
n is 0,1,2,3,4 mod 7
summitwei
2017-02-16 21:27:44
n = 4, 3, 2, 1, 0 mod 7
n = 4, 3, 2, 1, 0 mod 7
copeland
2017-02-16 21:27:47
The numerator is a multiple if 7 when $n\equiv 0,1,2,3,4\pmod7$.
The numerator is a multiple if 7 when $n\equiv 0,1,2,3,4\pmod7$.
copeland
2017-02-16 21:27:53
We probably want to factor that denominator better. Since we know it's always divisible by 5, let's get that out of there.
We probably want to factor that denominator better. Since we know it's always divisible by 5, let's get that out of there.
copeland
2017-02-16 21:27:55
We want to find when\[5t=\frac{n(n-1)(n-2)(n-3)(n-4)}{7\cdot3^2\cdot2^5}\]is an integer.
We want to find when\[5t=\frac{n(n-1)(n-2)(n-3)(n-4)}{7\cdot3^2\cdot2^5}\]is an integer.
copeland
2017-02-16 21:28:09
The numerator is a multiple if 7 when $n\equiv 0,1,2,3,4\pmod7$.
The numerator is a multiple if 7 when $n\equiv 0,1,2,3,4\pmod7$.
copeland
2017-02-16 21:28:11
Now before we go too far down this path, what tool are we going to need to invoke on this problem?
Now before we go too far down this path, what tool are we going to need to invoke on this problem?
Derive_Foiler
2017-02-16 21:28:48
CRT seems nice and bashy
CRT seems nice and bashy
linqaszayi
2017-02-16 21:28:48
CRT
CRT
brainiac1
2017-02-16 21:28:48
crt?
crt?
duck_master
2017-02-16 21:28:48
Chinese Remainder Theorem!
Chinese Remainder Theorem!
nukelauncher
2017-02-16 21:28:48
Chinese Remainder Theorem
Chinese Remainder Theorem
summitwei
2017-02-16 21:28:48
chinese remainder theorem
chinese remainder theorem
GeronimoStilton
2017-02-16 21:28:48
CRT?
CRT?
copeland
2017-02-16 21:28:50
A number is divisible by the product of coprime numbers if and only if it is divisible by each of the factors. That's a trivial case of the Chinese Remainder Theorem. It also follows directly from the Fundamental Theorem of Arithmetic. It's also just painfully obvious.
A number is divisible by the product of coprime numbers if and only if it is divisible by each of the factors. That's a trivial case of the Chinese Remainder Theorem. It also follows directly from the Fundamental Theorem of Arithmetic. It's also just painfully obvious.
copeland
2017-02-16 21:28:58
So what's the product of those things in the denominator?
So what's the product of those things in the denominator?
brainiac1
2017-02-16 21:30:03
2016!
2016!
tdeng
2017-02-16 21:30:03
2016
2016
Slacker
2017-02-16 21:30:03
2016
2016
brainiac1
2017-02-16 21:30:03
2016
2016
abishek99
2017-02-16 21:30:03
2016
2016
duck_master
2017-02-16 21:30:03
2016 which was last year
2016 which was last year
GeronimoStilton
2017-02-16 21:30:03
2016!
2016!
BXU65
2017-02-16 21:30:03
2016!
2016!
copeland
2017-02-16 21:30:05
\begin{align*}7\cdot3^2\cdot2^5
&=7\cdot9\cdot2^5\\
&=63\cdot2^5\\
&=(2^6-1)2^5\\
&=2^{11}-2^5\\
&=2048-32\\
&=2016.
\end{align*}
\begin{align*}7\cdot3^2\cdot2^5
&=7\cdot9\cdot2^5\\
&=63\cdot2^5\\
&=(2^6-1)2^5\\
&=2^{11}-2^5\\
&=2048-32\\
&=2016.
\end{align*}
copeland
2017-02-16 21:30:08
Wow, unbelievable. Anyone feel like we're on the right track here?
Wow, unbelievable. Anyone feel like we're on the right track here?
GeronimoStilton
2017-02-16 21:30:27
Is that intentional?
Is that intentional?
tdeng
2017-02-16 21:30:27
Ye.
Ye.
mssmath
2017-02-16 21:30:27
yup
yup
BXU65
2017-02-16 21:30:27
This is nostalgic
This is nostalgic
Jfault
2017-02-16 21:30:27
no, we got 2016 instead of 2017 /s
no, we got 2016 instead of 2017 /s
copeland
2017-02-16 21:30:28
Yeah, retro.
Yeah, retro.
copeland
2017-02-16 21:30:33
We can figure out the exact fraction of the numbers from 2 to 2017 that make this product divisible by all of these factors!
We can figure out the exact fraction of the numbers from 2 to 2017 that make this product divisible by all of these factors!
copeland
2017-02-16 21:30:35
Now that we're reinvigorated, when is the numerator divisible by 9?
Now that we're reinvigorated, when is the numerator divisible by 9?
Jfault
2017-02-16 21:31:57
whenever there are two factors of 3 in it
whenever there are two factors of 3 in it
copeland
2017-02-16 21:32:03
When's that?
When's that?
yrnsmurf
2017-02-16 21:32:58
when n is not 5 or 8 mod 9
when n is not 5 or 8 mod 9
summitwei
2017-02-16 21:32:58
n is not 5 or 8 mod 9
n is not 5 or 8 mod 9
brainiac1
2017-02-16 21:32:58
n = 0, 1, 2, 3, 4, 6, 7 mod 9
n = 0, 1, 2, 3, 4, 6, 7 mod 9
BXU65
2017-02-16 21:32:58
n mod 9 is 0, 1, 2, 3, 4, 6, 7
n mod 9 is 0, 1, 2, 3, 4, 6, 7
copeland
2017-02-16 21:33:04
The product of 5 consecutive numbers is divisible by 9 when there are two threes or one nine. Therefore a product of 5 consecutive numbers is not divisible by 9 when the middle number is a multiple of 3 but not a multiple of nine. The middle number is $n-2$, so $n-2$ can't be 3 or 6.
The product of 5 consecutive numbers is divisible by 9 when there are two threes or one nine. Therefore a product of 5 consecutive numbers is not divisible by 9 when the middle number is a multiple of 3 but not a multiple of nine. The middle number is $n-2$, so $n-2$ can't be 3 or 6.
copeland
2017-02-16 21:33:11
The numerator is divisible by 9 when $n\equiv0,1,2,3,4,6,7\pmod9$.
The numerator is divisible by 9 when $n\equiv0,1,2,3,4,6,7\pmod9$.
copeland
2017-02-16 21:33:19
Now let's figure out when the numerator is divisible by 32. (Yeah, I know - I didn't write the problem.)
Now let's figure out when the numerator is divisible by 32. (Yeah, I know - I didn't write the problem.)
copeland
2017-02-16 21:33:36
Let's make a table of $n\pmod{32}$ and the highest power of two that must divide $n(n-1)(n-2)(n-3)(n-4)$. We'll call that $V(n)$.
Let's make a table of $n\pmod{32}$ and the highest power of two that must divide $n(n-1)(n-2)(n-3)(n-4)$. We'll call that $V(n)$.
copeland
2017-02-16 21:33:38
Actually, let's start with $v(n)$, just the number of 2s dividing $n$.
Actually, let's start with $v(n)$, just the number of 2s dividing $n$.
copeland
2017-02-16 21:34:05
\[\begin{array}{c|c}
n\pmod{32}&-4&-3&-2&-1&0&1&2&3&4&5\\
\hline
v(n)&2&0&1&0&5&0&1&0&2&0\\
V(n)& & & & &\color{green}{8}&
\end{array}\]
\[\begin{array}{c|c}
n\pmod{32}&-4&-3&-2&-1&0&1&2&3&4&5\\
\hline
v(n)&2&0&1&0&5&0&1&0&2&0\\
V(n)& & & & &\color{green}{8}&
\end{array}\]
copeland
2017-02-16 21:34:21
$V(0)=v(-4)+v(-3)+v(-2)+v(-1)+v(0).$
$V(0)=v(-4)+v(-3)+v(-2)+v(-1)+v(0).$
copeland
2017-02-16 21:34:30
We don't care about the values of $V$ for negative $n$. What are the entries for $n=1$ through $n=5$?
We don't care about the values of $V$ for negative $n$. What are the entries for $n=1$ through $n=5$?
brainiac1
2017-02-16 21:36:23
6, 7, 6, 8, 3
6, 7, 6, 8, 3
BXU65
2017-02-16 21:36:23
6, 7, 6, 8, 3
6, 7, 6, 8, 3
xayy
2017-02-16 21:36:23
6,7,6,8,3
6,7,6,8,3
yrnsmurf
2017-02-16 21:36:23
67683
67683
copeland
2017-02-16 21:36:26
\begin{align*}
V(1)&=v(-3)+v(-2)+v(-1)+v(0)+v(1)\\
V(2)&=v(-2)+v(-1)+v(0)+v(1)+v(2)\\
V(3)&=v(-1)+v(0)+v(1)+v(2)+v(3)\\
V(4)&=v(0)+v(1)+v(2)+v(3)+v(4)\\
V(5)&=v(1)+v(2)+v(3)+v(4)+v(5)
\end{align*}
\begin{align*}
V(1)&=v(-3)+v(-2)+v(-1)+v(0)+v(1)\\
V(2)&=v(-2)+v(-1)+v(0)+v(1)+v(2)\\
V(3)&=v(-1)+v(0)+v(1)+v(2)+v(3)\\
V(4)&=v(0)+v(1)+v(2)+v(3)+v(4)\\
V(5)&=v(1)+v(2)+v(3)+v(4)+v(5)
\end{align*}
copeland
2017-02-16 21:36:27
\[\begin{array}{c|c}
n\pmod{32}&-4&-3&-2&-1&0&1&2&3&4&5\\
\hline
v(n)&2&0&1&0&5&0&1&0&2&0\\
V(n)& & & & &\color{green}{8}&\color{green}{6}&\color{green}{7}&\color{green}{6}&\color{green}{8}&3\\
\end{array}\]
\[\begin{array}{c|c}
n\pmod{32}&-4&-3&-2&-1&0&1&2&3&4&5\\
\hline
v(n)&2&0&1&0&5&0&1&0&2&0\\
V(n)& & & & &\color{green}{8}&\color{green}{6}&\color{green}{7}&\color{green}{6}&\color{green}{8}&3\\
\end{array}\]
Jfault
2017-02-16 21:36:32
why is 8 in green
why is 8 in green
copeland
2017-02-16 21:36:38
Yeah, what is that green stuff about?
Yeah, what is that green stuff about?
duck_master
2017-02-16 21:37:18
greater than 5
greater than 5
brainiac1
2017-02-16 21:37:18
it is greater than 5, so the numerator is divisible by 32
it is greater than 5, so the numerator is divisible by 32
BXU65
2017-02-16 21:37:18
the values that are divisible by 32
the values that are divisible by 32
GeronimoStilton
2017-02-16 21:37:18
Because it works for a possible value of $n$.
Because it works for a possible value of $n$.
Derive_Foiler
2017-02-16 21:37:18
all above 5. good
all above 5. good
copeland
2017-02-16 21:37:19
I marked all the numbers where $V(n)\geq5$ since those are the values of $n$ where the numerator is divisible by 35 (contains 5 powers of 2).
I marked all the numbers where $V(n)\geq5$ since those are the values of $n$ where the numerator is divisible by 35 (contains 5 powers of 2).
copeland
2017-02-16 21:37:21
What are the next two values of $V(n)$?
What are the next two values of $V(n)$?
copeland
2017-02-16 21:37:23
\[\begin{array}{c|c}
n\pmod{32}&-4&-3&-2&-1&0&1&2&3&4&5&6&7\\
\hline
v(n)&2&0&1&0&5&0&1&0&2&0&1&0\\
V(n)& & & & &\color{green}{8}&\color{green}{6}&\color{green}{7}&\color{green}{6}&\color{green}{8}&3\\
\end{array}\]
\[\begin{array}{c|c}
n\pmod{32}&-4&-3&-2&-1&0&1&2&3&4&5&6&7\\
\hline
v(n)&2&0&1&0&5&0&1&0&2&0&1&0\\
V(n)& & & & &\color{green}{8}&\color{green}{6}&\color{green}{7}&\color{green}{6}&\color{green}{8}&3\\
\end{array}\]
BXU65
2017-02-16 21:38:07
4, 3
4, 3
brainiac1
2017-02-16 21:38:07
4, 3
4, 3
yrnsmurf
2017-02-16 21:38:07
43
43
GeronimoStilton
2017-02-16 21:38:07
$4, 3$
$4, 3$
xayy
2017-02-16 21:38:07
4,3
4,3
copeland
2017-02-16 21:38:09
Next we have
Next we have
copeland
2017-02-16 21:38:10
\[\begin{array}{c|c}
n\pmod{32}&-4&-3&-2&-1&0&1&2&3&4&5&6&7&8&9&10&11&12&13\\
\hline
v(n)&2&0&1&0&5&0&1&0&2&0&1&0&3&0&1&0&2&0\\
V(n)& & & & &\color{green}{8}&\color{green}{6}&\color{green}{7}&\color{green}{6}&\color{green}{8}&3&4&3\\
\end{array}\]
\[\begin{array}{c|c}
n\pmod{32}&-4&-3&-2&-1&0&1&2&3&4&5&6&7&8&9&10&11&12&13\\
\hline
v(n)&2&0&1&0&5&0&1&0&2&0&1&0&3&0&1&0&2&0\\
V(n)& & & & &\color{green}{8}&\color{green}{6}&\color{green}{7}&\color{green}{6}&\color{green}{8}&3&4&3\\
\end{array}\]
copeland
2017-02-16 21:38:11
And the next 6 terms?
And the next 6 terms?
BXU65
2017-02-16 21:39:25
6, 4, 5, 4, 6, 3
6, 4, 5, 4, 6, 3
xayy
2017-02-16 21:39:25
6,4,5,4,6,3
6,4,5,4,6,3
brainiac1
2017-02-16 21:39:25
6, 4, 5, 4, 6, 3
6, 4, 5, 4, 6, 3
duck_master
2017-02-16 21:39:25
6,4,5,4,6,3
6,4,5,4,6,3
yrnsmurf
2017-02-16 21:39:25
645463
645463
Reef334
2017-02-16 21:39:25
6,4,5,4,6,3
6,4,5,4,6,3
rt03
2017-02-16 21:39:25
6,4,5,4,6,3
6,4,5,4,6,3
copeland
2017-02-16 21:39:28
\[\begin{array}{c|c}
n\pmod{32}&-4&-3&-2&-1&0&1&2&3&4&5&6&7&8&9&10&11&12&13\\
\hline
v(n)&2&0&1&0&5&0&1&0&2&0&1&0&3&0&1&0&2&0\\
V(n)& & & & &\color{green}{8}&\color{green}{6}&\color{green}{7}&\color{green}{6}&\color{green}{8}&3&4&3&\color{green}{6}&4&\color{green}{5}&4&\color{green}{6}&3\\
\end{array}\]
\[\begin{array}{c|c}
n\pmod{32}&-4&-3&-2&-1&0&1&2&3&4&5&6&7&8&9&10&11&12&13\\
\hline
v(n)&2&0&1&0&5&0&1&0&2&0&1&0&3&0&1&0&2&0\\
V(n)& & & & &\color{green}{8}&\color{green}{6}&\color{green}{7}&\color{green}{6}&\color{green}{8}&3&4&3&\color{green}{6}&4&\color{green}{5}&4&\color{green}{6}&3\\
\end{array}\]
copeland
2017-02-16 21:39:29
OK, I think we get the idea. I'll give you a few more, clipping off those negatives for now:
OK, I think we get the idea. I'll give you a few more, clipping off those negatives for now:
copeland
2017-02-16 21:39:30
\[\begin{array}{c|c}
n\pmod{32}&0&1&2&3&4&5&6&7&8&9&10&11&12&13&14&15&16&17&18&19&20\\
\hline
v(n)&5&0&1&0&2&0&1&0&3&0&1&0&2&0&1&0&4&0&1&0&2\\
V(n)&\color{green}{8}&\color{green}{6}&\color{green}{7}&\color{green}{6}&\color{green}{8}&3&4&3&\color{green}{6}&4&\color{green}{5}&4&\color{green}{6}&3&4&3&\color{green}{7}&\color{green}{5}&\color{green}{6}&\color{green}{5}&\color{green}{7}\\
\end{array}\]
\[\begin{array}{c|c}
n\pmod{32}&0&1&2&3&4&5&6&7&8&9&10&11&12&13&14&15&16&17&18&19&20\\
\hline
v(n)&5&0&1&0&2&0&1&0&3&0&1&0&2&0&1&0&4&0&1&0&2\\
V(n)&\color{green}{8}&\color{green}{6}&\color{green}{7}&\color{green}{6}&\color{green}{8}&3&4&3&\color{green}{6}&4&\color{green}{5}&4&\color{green}{6}&3&4&3&\color{green}{7}&\color{green}{5}&\color{green}{6}&\color{green}{5}&\color{green}{7}\\
\end{array}\]
copeland
2017-02-16 21:39:37
This sure isn't getting any more fun.
This sure isn't getting any more fun.
copeland
2017-02-16 21:39:39
Actually, I think we needed to do all that work, but maybe we can stop now, why?
Actually, I think we needed to do all that work, but maybe we can stop now, why?
brainiac1
2017-02-16 21:40:29
there's a pattern?
there's a pattern?
copeland
2017-02-16 21:40:32
It's patternish. How can we talk about the top half of the numbers?
It's patternish. How can we talk about the top half of the numbers?
Derive_Foiler
2017-02-16 21:40:35
it'll repeat to some extent
it'll repeat to some extent
copeland
2017-02-16 21:40:46
Kinda. Does $v(n)$ repeat?
Kinda. Does $v(n)$ repeat?
brainiac1
2017-02-16 21:41:11
every 32...
every 32...
GeronimoStilton
2017-02-16 21:41:11
Yes, modulo 32.
Yes, modulo 32.
copeland
2017-02-16 21:41:15
Yeah, that's true.
Yeah, that's true.
copeland
2017-02-16 21:41:24
Does it have some smaller similarity?
Does it have some smaller similarity?
Derive_Foiler
2017-02-16 21:41:52
and doesn't it mirror at 16?
and doesn't it mirror at 16?
copeland
2017-02-16 21:41:53
Oh, it does "mirror"! There's reflection symmetry. Actually, the reflection at 2 is a little easier to see.
Oh, it does "mirror"! There's reflection symmetry. Actually, the reflection at 2 is a little easier to see.
BXU65
2017-02-16 21:41:59
the number of factors of two has a pattern
the number of factors of two has a pattern
linqaszayi
2017-02-16 21:41:59
mod 16?
mod 16?
copeland
2017-02-16 21:42:03
Also, $v(n)$ mostly repeats mod 16 except $v(16)$ and $v(0)$ are different.
Also, $v(n)$ mostly repeats mod 16 except $v(16)$ and $v(0)$ are different.
copeland
2017-02-16 21:42:10
So there are two things that work here.
So there are two things that work here.
copeland
2017-02-16 21:42:12
We could notice that $k$ and $-k$ have the same number of factors so the number of factors of 2 in $n(n-1)(n-2)(n-3)(n-4)$ needs to be symmetric about $n=2$ and $n=18$. So we can look at $n=2$ and $n=18$ and we can double all the rest to get our final count.
We could notice that $k$ and $-k$ have the same number of factors so the number of factors of 2 in $n(n-1)(n-2)(n-3)(n-4)$ needs to be symmetric about $n=2$ and $n=18$. So we can look at $n=2$ and $n=18$ and we can double all the rest to get our final count.
copeland
2017-02-16 21:42:21
Alternatively, we could notice that everything repeats modulo 16 except that $n=0$ through $n=4$ get an extra power of 2. Therefore, again we can "double" the number of successes up to edge cases.
Alternatively, we could notice that everything repeats modulo 16 except that $n=0$ through $n=4$ get an extra power of 2. Therefore, again we can "double" the number of successes up to edge cases.
copeland
2017-02-16 21:42:30
How many of the residues modulo 32 solve $n(n-1)(n-2)(n-3)(n-4)\equiv0\pmod{32}$?
How many of the residues modulo 32 solve $n(n-1)(n-2)(n-3)(n-4)\equiv0\pmod{32}$?
mathman2048
2017-02-16 21:43:28
16
16
yrnsmurf
2017-02-16 21:43:28
16
16
BXU65
2017-02-16 21:43:28
16
16
abishek99
2017-02-16 21:43:28
16
16
GeronimoStilton
2017-02-16 21:43:28
$16$?
$16$?
copeland
2017-02-16 21:43:30
From 3 to 17 there are 7 green numbers. Doubling that and adding 2 for $n=2$ and $n=18$ gives us $\dfrac{16}{32}$ of the numbers are multiples of 32.
From 3 to 17 there are 7 green numbers. Doubling that and adding 2 for $n=2$ and $n=18$ gives us $\dfrac{16}{32}$ of the numbers are multiples of 32.
copeland
2017-02-16 21:43:36
What fraction of the numbers from 1 to 2017 make $\dfrac{n(n-1)(n-2)(n-3)(n-4)}{7\cdot9\cdot32}$ and integer?
What fraction of the numbers from 1 to 2017 make $\dfrac{n(n-1)(n-2)(n-3)(n-4)}{7\cdot9\cdot32}$ and integer?
copeland
2017-02-16 21:44:11
Or at least, how do we compute the fraction?
Or at least, how do we compute the fraction?
ninjataco
2017-02-16 21:45:23
multiply all the fractions from all of the mods
multiply all the fractions from all of the mods
duck_master
2017-02-16 21:45:23
More Chinese Remainder Theorem!
More Chinese Remainder Theorem!
BXU65
2017-02-16 21:45:23
1/2 * 7/9 * 5/7 which is 5/18
1/2 * 7/9 * 5/7 which is 5/18
GeronimoStilton
2017-02-16 21:45:23
$\frac{7 \cdot 16 \cdot 5}{2016}$
$\frac{7 \cdot 16 \cdot 5}{2016}$
Reef334
2017-02-16 21:45:23
5/7*7/9*16/32
5/7*7/9*16/32
yrnsmurf
2017-02-16 21:45:23
16*7*5
16*7*5
copeland
2017-02-16 21:45:26
Well, $\dfrac57$ of them are divisible by 7, $\dfrac79$ of them are divisible by 9, and $\dfrac{16}{32}$ of them are divisible by 32.
Well, $\dfrac57$ of them are divisible by 7, $\dfrac79$ of them are divisible by 9, and $\dfrac{16}{32}$ of them are divisible by 32.
copeland
2017-02-16 21:45:29
Those are independent by the Painfully Obvious Lemma (though it feels less obvious than painful now), so the number of integers is\[\dfrac57\cdot\dfrac79\cdot\dfrac{20}{32}=\dfrac{35\cdot16}{2016}=\dfrac{560}{2016}.\]
Those are independent by the Painfully Obvious Lemma (though it feels less obvious than painful now), so the number of integers is\[\dfrac57\cdot\dfrac79\cdot\dfrac{20}{32}=\dfrac{35\cdot16}{2016}=\dfrac{560}{2016}.\]
copeland
2017-02-16 21:45:37
560 is not one of the answers!
560 is not one of the answers!
brainiac1
2017-02-16 21:46:24
it's from 9 to 2017...
it's from 9 to 2017...
linqaszayi
2017-02-16 21:46:24
$n\ge 9$
$n\ge 9$
celestialphoenix3768
2017-02-16 21:46:24
must subtract 3 for working 2-9
must subtract 3 for working 2-9
shakeNbake
2017-02-16 21:46:24
then assume u miscounted by 3 and get (D)
then assume u miscounted by 3 and get (D)
duck_master
2017-02-16 21:46:24
did you talk about 1,2,3,4,5,6,7,8,9,2017?
did you talk about 1,2,3,4,5,6,7,8,9,2017?
copeland
2017-02-16 21:46:27
Oh, we're not allowed to have $n=2$ through $n=8$. Too bad they didn't save this problem until 2024. (In fact, we can tell because the denominator is also wrong.)
Oh, we're not allowed to have $n=2$ through $n=8$. Too bad they didn't save this problem until 2024. (In fact, we can tell because the denominator is also wrong.)
copeland
2017-02-16 21:46:30
Remember, we're looking at the 2016 numbers 2-2017.
Remember, we're looking at the 2016 numbers 2-2017.
copeland
2017-02-16 21:46:34
Those first 7 values of $n$ could have worked or not. If they all were "good", then the answer is $560-7=553$. If they all failed, the answer is 560.
Those first 7 values of $n$ could have worked or not. If they all were "good", then the answer is $560-7=553$. If they all failed, the answer is 560.
copeland
2017-02-16 21:46:34
So. . . .
So. . . .
BXU65
2017-02-16 21:47:22
it needs to be 557 or D
it needs to be 557 or D
GeronimoStilton
2017-02-16 21:47:22
The answer's 557, D.
The answer's 557, D.
summitwei
2017-02-16 21:47:22
somewhere in between, so d
somewhere in between, so d
duck_master
2017-02-16 21:47:22
*ahem* look at the answer choices!
*ahem* look at the answer choices!
copeland
2017-02-16 21:47:24
The only answer in that range is (D) $\boxed{557}$.
The only answer in that range is (D) $\boxed{557}$.
copeland
2017-02-16 21:48:18
So, I guess we're obligated to figure out which three of the first 8 gave bad integers. AND, we're probably obligated to argue that we can make all of these.
So, I guess we're obligated to figure out which three of the first 8 gave bad integers. AND, we're probably obligated to argue that we can make all of these.
copeland
2017-02-16 21:48:22
First, can we pick a set of $t=\dfrac{n(n-1)(n-2)(n-3)(n-4)}{8\cdot7\cdot6\cdot5\cdot3\cdot2}$ different teams from a group of $n$ people?
First, can we pick a set of $t=\dfrac{n(n-1)(n-2)(n-3)(n-4)}{8\cdot7\cdot6\cdot5\cdot3\cdot2}$ different teams from a group of $n$ people?
copeland
2017-02-16 21:49:12
How many total possible teams are there?
How many total possible teams are there?
GeronimoStilton
2017-02-16 21:49:49
Yes, if $n \ge 4$
Yes, if $n \ge 4$
BXU65
2017-02-16 21:49:49
n choose 5
n choose 5
duck_master
2017-02-16 21:49:49
$\binom{n}{5}$
$\binom{n}{5}$
copeland
2017-02-16 21:49:51
Since \[\dbinom n5=\dfrac{n(n-1)(n-2)(n-3)(n-4)}{5\cdot4\cdot3\cdot2\cdot1}\] is way larger than $t$, we can pick any set of $t$ teams and be done with it (when $t$ is an integer).
Since \[\dbinom n5=\dfrac{n(n-1)(n-2)(n-3)(n-4)}{5\cdot4\cdot3\cdot2\cdot1}\] is way larger than $t$, we can pick any set of $t$ teams and be done with it (when $t$ is an integer).
copeland
2017-02-16 21:49:56
Second, let's see how many of the first 8 succeed:
Second, let's see how many of the first 8 succeed:
copeland
2017-02-16 21:49:57
\[\begin{array}{c|c}
n&2&3&4&5&6&7&8\\
\hline
V(n)&\color{green}{7}&\color{green}{6}&\color{green}{8}&3&4&3&\color{green}{6}\\
n\pmod{7}&\color{green}{2}&\color{green}{3}&\color{green}{4}&5&6&\color{green}{0}&\color{green}{1}\\
n\pmod{9}&\color{green}{2}&\color{green}{3}&\color{green}{4}&5&\color{green}{6}&\color{green}{7}&8\\
\end{array}\]
\[\begin{array}{c|c}
n&2&3&4&5&6&7&8\\
\hline
V(n)&\color{green}{7}&\color{green}{6}&\color{green}{8}&3&4&3&\color{green}{6}\\
n\pmod{7}&\color{green}{2}&\color{green}{3}&\color{green}{4}&5&6&\color{green}{0}&\color{green}{1}\\
n\pmod{9}&\color{green}{2}&\color{green}{3}&\color{green}{4}&5&\color{green}{6}&\color{green}{7}&8\\
\end{array}\]
yrnsmurf
2017-02-16 21:50:14
1,2,3,4 work 5,6,7 don't
1,2,3,4 work 5,6,7 don't
copeland
2017-02-16 21:50:15
The numbers 2, 3, and 4 are the omitted ones so $560-3=557$ is indeed the right answer.
The numbers 2, 3, and 4 are the omitted ones so $560-3=557$ is indeed the right answer.
copeland
2017-02-16 21:50:24
And we win?
And we win?
brainiac1
2017-02-16 21:50:48
yup?
yup?
copeland
2017-02-16 21:50:56
Enthusiasm and delight.
Enthusiasm and delight.
copeland
2017-02-16 21:51:11
Well, That's it for the Math Jam.
Well, That's it for the Math Jam.
copeland
2017-02-16 21:51:18
Thanks everyone for coming!
Thanks everyone for coming!
GeronimoStilton
2017-02-16 21:51:21
Bye!!!
Bye!!!
brainiac1
2017-02-16 21:51:21
Thanks!
Thanks!
copeland
2017-02-16 21:51:23
AoPS is running our weekend Special AIME Problem Seminar next weekend, Feb 25 and 26, from 3:30-6:30 PM Eastern each day. More information is at http://artofproblemsolving.com/school/course/catalog/maa-aime-special.
AoPS is running our weekend Special AIME Problem Seminar next weekend, Feb 25 and 26, from 3:30-6:30 PM Eastern each day. More information is at http://artofproblemsolving.com/school/course/catalog/maa-aime-special.
copeland
2017-02-16 21:51:23
Please join us again on March 9 and 24 when we will be discussing the AIME I and II contests.
Please join us again on March 9 and 24 when we will be discussing the AIME I and II contests.
BXU65
2017-02-16 21:52:07
What's the best way to prepare for AIME
What's the best way to prepare for AIME
copeland
2017-02-16 21:52:09
Buy all the AoPS books and take classes, too.
Buy all the AoPS books and take classes, too.
copeland
2017-02-16 21:52:31
Also, check out our forums. This question is on there a billion times and a lot of the real answers are very good.
Also, check out our forums. This question is on there a billion times and a lot of the real answers are very good.
copeland
2017-02-16 21:52:49
Alright, I'm going to close everything down now. Thanks for coming!
Alright, I'm going to close everything down now. Thanks for coming!
BXU65
2017-02-16 21:53:04
Ok Thanks Bye!
Ok Thanks Bye!
swimmerstar
2017-02-16 21:53:04
Thanks
Thanks
ImpossibleCube
2017-02-16 21:53:04
Thanks!
Thanks!
anixsarkar
2017-02-16 21:53:04
thank you
thank you
Copyright © 2024 AoPS Incorporated. This page is copyrighted material. You can view and print this page for your own use, but you cannot share the contents of this file with others.